Microbiology Test For Examanswer by Assel Maam

You might also like

Download as pdf or txt
Download as pdf or txt
You are on page 1of 157

lOMoARcPSD|19252957

Microbiology test for exam(answer by assel ma'am)

General medicine (Kyrgyz State Medical Academy)

Studocu is not sponsored or endorsed by any college or university


Downloaded by AMAN SINGH (amansinghgkp8@gmail.com)
lOMoARcPSD|19252957

EXAMINATION TESTS OF MICROBIOLOGY VIROLOGY AND


IMMUNOLOGY DISCIPLINE
FOR FACULTY OF GENERAL MEDICINE FOR FOREIGN CITIZENS

Basic bacteriology
1. Who first designed the microscope, saw and sketched microbes?
A. Louis Pasteur
B. Robert Koch
C. Ilya Mechnikov
D. Dmitri Ivanovsky
E. Antonie Philips van Leeuwenhoek
2. Without which structures, bacteria cannot carry out their activities?
A. Capsule
B. Spors
C. Volutine granules
D. Nucleoid
E. Flagella
3. What structure determines the shape of a bacterial cell?
A. Cytoplasmic membrane
B. Capsid
C. Capsule
D. Spore
E. Cell wall
4. Bacteria as a unicellular microbes, that are without a nuclear membrane,
belong to which Kingdoms?
A. Eukaryotes
B. Priones
C. Prokaryotes
D. Viruses
E. Fungi
5. What is the significant structural component of a bacterial cell?
A. Capsule
B. Flagella
C. Spore
D. Cell wall
E. Volutine granules
6. From what structural element of bacteria does Gram stain depend on?
A. Structure of plasma membrane
B. Сomposition of cytoplasme
C. Structure of cell wall
D. Arraignment of nucleoid
E. Сomposition of Volutine granules

7. The structure of what morphological components determines Gram staining?

Downloaded by AMAN SINGH (amansinghgkp8@gmail.com)


lOMoARcPSD|19252957

A. Cytoplasm
B. Capsule
C. Cell wall
D. Plasma membrane
E. Volutine granules
8. Which step or dye does not use in Gram staining?
A. Alcohol discoloration
B. Fuchsin red
C. Crystal violet
D. Heating over the spirit lamp
E. Iodine solution
9. What is the reason for the acid fastness of bacteria?
A. Presence of carbohydrates
B. Presence of glycogen
C. Presence of proteins
D. Presence of lipids
E. Presence of peptidoglycan
10. What is the difference of cell wall structure between gram positive and gram
negative bacteria?
A. One layer of peptidoglycan
B. Lipopolysaccharide
C. Teichoic acids
D. Mitochondria
E. Mesosomes
11. What types of nutrition do have pathogenic bacteria?
A. Metatrophs
B. Autotrophs
C. Paratrophs
D. Heterotrophs
E. Auxotrophs
12. What mechanism is carried out with the active transport of nutrients into the
cell?
A. Without cleavage of the substrate
B. None of listed bellow
C. Without energy costs
D. By concentration of gradient
E. Against a concentration gradient
13. What is sterilization method completely destroys microbes in the material?
A. Autoclaving
B. Pasteurization
C. Filtration
D. Tindalization
E. Drying
14. Which structure does determine the ability of bacteria to attach to the surface of
host cells?

Downloaded by AMAN SINGH (amansinghgkp8@gmail.com)


lOMoARcPSD|19252957

A. Cell wall
B. Mesosomes
C. Fimbria
D. Flagella
E. Plasma membrane
15. What is the function of nucleoid?
A. To keep the shape of bacteria
B. To keep osmotic pressure
C. To keep hereditary information
D To keep cell’s metabolism
E. To keep protein synthesis
16. What is the function of cell wall?
A. Formative
B. Spore-forming
C. Capsule-forming
D. Enzyme-forming
E. Energy-generating
17. What is the importance of chemical composition of the cell wall?
A. Protein Synthesizing System
B. Contains ribosomes for protein synthesis
C. This structure is the same for all bacteria
D. Keeps hereditary information
E. It is responsible for Gram staining

18. What is the function of cytoplasmic membrane in bacteria?


A. Gives shape to cells
B. Involved in the transport of substances
C. Participates in protein synthesis
D. Causes cell sensitization.
E. Causes cell chemotaxis

19. Which is always present in bacteria ?


A. Cell wall
B. Cytoplasmic membrane
C. Mitochondria
D. Nucleoli
E. Volutin granules

20. Teichoic acids:

A. Are found in the walls of many gram-positive bacteria


B. Make up the outer wall of bacteria

Downloaded by AMAN SINGH (amansinghgkp8@gmail.com)


lOMoARcPSD|19252957

C. Provide receptors for phages


D. Influence the permeability of membrane
E. All are true

21. Mesosomes in bacteria are functional unit for :

A. Lipid storage
B.Protein synthesis
C.Respiratory enzymes
D.None
E. All
22. Essential structures for bacteria:

A. Capsule
B. Spores
C. Volutin granules
D. Nucleoid
E. Flagella

23. The presence of the cell wall is determined by:

A. Fluorescent microscopy
B. The “crushed drop” method
C. By the “thick drop” method
D. Ultracentrifugation
E. Plasmolite

24. The shape of the bacterial cell is determined by the structure:

A. Cytoplasmic membrane
B. Capsid
C. Capsules
D. Spores
E. Cell wall

25. Bacteria refer to:

A. Eukaryotes
B. Prions

Downloaded by AMAN SINGH (amansinghgkp8@gmail.com)


lOMoARcPSD|19252957

C. Prokaryotes
D. All three
E. None of them

26. A prokaryotic cell has:

A. Golgi Apparatus
B. Mitochondria
C. Morphologically formed core
D. Mesosomes
E. Nuclear membrane

27. The morphology of the bacteria is studied:

A. Dry Microscope System


B. In unpainted preparations
C. Immersion microscopy
D. With a small increase
E. None of the above

28. Essential structural component of the bacterial cell:

A. Capsule
B. Flagella
C. Spores
D. Cell wall
E. Volutin granules

29. Gram stain depends on:

A. Structures of the cytoplasmic membrane


B. The composition of the cytoplasm
C. Cell wall structures
D. Core Locations
E. From all listed signs

30. What are the morphological structures of bacteria cause gram stain ?

A. Cytoplasm

Downloaded by AMAN SINGH (amansinghgkp8@gmail.com)


lOMoARcPSD|19252957

B. Capsule
C. Cell wall
D. Cytoplasmic membrane
E. Volutin granule

31. The acid resistance of the microbial cell is associated with the presence of a
large number:

A. Carbohydrate
B. Glycogen
C. Protein
D. Lipids
E. Tryptophan

32. The cell wall of gram-positive bacteria contains:

A. Monolayer Peptidoglycan
B. Lipopolysaccharide
C. Teichoic Acids
D. Mitochondria
E. Mesosomes

33. The cell wall of gram-negative microbes contains:

A. Multilayer peptidoglycan
B. Volutin granule
C. Lipopolysaccharide
D. Mesosomes
E. Teichoic Acids

34. Microorganisms lose the cell wall under the action of:

A. Bacteriophage
B. Interferon
C. Immunoglobulin
D. Lysozyme
E. Nucleases

Downloaded by AMAN SINGH (amansinghgkp8@gmail.com)


lOMoARcPSD|19252957

35. A terminal spore resembling a drumstick is characteristic of:

A. Cl. perfringens
B. Cl. novyi
C. Cl. tetani
D. Cl. botulinum
E. Cl. septicum

36. Spiral forms have:

A. Chlamydia
B. Borrelia
C. Cocci
D. Bacteria
E. Mycoplasma

37. Spore-forming microorganisms of a spindle-shaped form are called:

A. Bacilli
B. Bacteria
C. Bipolar
D. Clostridia
E. Rickettsia

38. Which one of the following bacteria is cell wall deficient?

A. Escherichia coli
B. Mycoplasma
C. Salmonella typhi
D. Treponema pallidum
E. Clostridium botulinum

39. About Volutine granule is true except:

A. Available from Corynebacterium diphtheria


B. Participate in sporulation
C. Storage of nutrients

Downloaded by AMAN SINGH (amansinghgkp8@gmail.com)


lOMoARcPSD|19252957

D. Composition - polyphosphate
E. For tinctorial properties - metachromatic

40. Volutin granule according to the author are:

A. Lipsha
B. Babes-Ernst
C. Babesha-Negri
D. Pashena
E. Guarnieri

41. Generation time of Mycobacterium tuberculosis is about?

A. 20 seconds
B. 20 minutes
C. 20 hours
D. 20 days
E. 2 weeks

42. Active transport of nutrients into the cell goes:

A. Without splitting the substrate


B. None of the above
C. Without energy
D. By concentration gradient
E. Against concentration gradient

43. Antigenic and biochemical properties of bacteria help determine:

A. Morphology of microorganisms
B. Species and intraspecific differences
C. Antibiotic Sensitivity
D. Phagolability
E. Tinctorial properties

44. Prokaryotes are characterized by the presence of:

A. Separated core

Downloaded by AMAN SINGH (amansinghgkp8@gmail.com)


lOMoARcPSD|19252957

B. Disjunctive reproduction
C. Nuclear membrane
D. Mitochondria
E. Cytoplasmic membrane

45. The nucleoid of bacteria:

A. Compact
B. Has a nuclear envelope
C. Is a plasmid
D. Double stranded circular DNA
E. Single-stranded RNA

46. The function of the nucleoid:

A.Synthesis of proteins
B. Regulation of osmotic pressure
C. Preservation of hereditary information
D. Energetic cell metabolism
E. Production of pathogenicity factors

47. Cell wall function:

A. Gives bacteria a specific shape


B. Does not contain peptidoglycan.
C. Participates in the synthesis of vitamins
D. Is vital structure
E. Participates in oxygenic processes

48. Bacteria cell wall:

A. Is a protein synthesis system of the cell


B. Contains ribosomes for protein synthesis
C. Structure and chemical composition is the same for all bacteria.
D. Saves Genetic Information
E . Causes the relation to coloring across

49. Structures required for L-forms of bacteria:

Downloaded by AMAN SINGH (amansinghgkp8@gmail.com)


lOMoARcPSD|19252957

A. Cell wall
B. Flagella
C. Capsule
D. Cytoplasmic membrane
E . Plasmids

50. Bacteria without a cell wall:

A. Rickettsia
B. Spirochetes
C. Actinomycetes
D. Chlamydia
E. Mycoplasma

51. Bean-shaped diplococci belong to the genus:

A. Mycobacterium
B. Treponem
C. Neisseria
D. Chlamydia
E. Leptospira

52. Lipopolysaccharides contained in the cell wall of bacteria are:

A. Exotoxins
B. Endotoxins
C. Enzymes
D. Electrolytes
E. Products of metabolism

53. Lipopolysaccharide cell wall:

A. Does not possess antigenic and toxic properties


B. Gives stiffness and elasticity.
C. Activates the complement
D. Is a strong immunogenic
E. Contained in gram-positive bacteria

Downloaded by AMAN SINGH (amansinghgkp8@gmail.com)


lOMoARcPSD|19252957

54. The cytoplasmic membrane of bacteria:

A. Defines cell shape


B. Participates in the transport of substances
C. Is a protein synthesizing system
D. Causes cell sensitization
E. Is a chemotaxis factor

55. The cytoplasmic membrane is:

A. Peptidoglycan consisting of parallel polysaccharide chains


B. The pronounced mucous layer covering the cell wall
C. Double phospholipid layer permeated with globulins
D. Inhibitor of bacterial cell wall synthesis
E. Complex nucleoprotein

56. The value of the cytoplasmic membrane of bacteria:


A. Has selective permeability
B. Stores the hereditary information of the bacterial cell
C. Participates in conjugation
D. Limits the core
E. Regulates the immune response

57. Bacterial cytoplasm:

A. Consists of mitochondria
B. Contains differentiated core
C. It is a complex colloidal system.
D. Does not contain ribosomes
E. Enhances virulence

58. Bacterial mesosomes:

A. Participate in cell division


B. Derived cell wall
C. Not bound to nucleoid
D. Are equivalent of nucleus
E. Strengthen biochemical activity

Downloaded by AMAN SINGH (amansinghgkp8@gmail.com)


lOMoARcPSD|19252957

59. The role of the capsule in the functioning of bacteria:

A. Is a required structural component of the cell


B. Is not strengthening virulence
C. Activate phagocytosis
D. Is an osmotic barrier
E. Strengthen the protective factors of the microorganism

60. The bacterial capsule is characterized by:

A. Easy to stain
B. High lipid content
C. Antigen specificity
D. Acid Resistance
E. Oncogenicity

61. The conditions stimulating the capsulation of bacteria is their growth in:

A. In an animal organism
B. On blood agar
C. On synthetic medium
D. On a high carbohydrate environment
E. At low temperatures

62. The bacterial capsule is detected:

A. In a phase contrast microscope


B. Staining by Albert
C. Staining by Indian ink
D. Fluorescent microscopy
E. In the experience of plasmolysis cells

63. Capsules in the body are formed by pathogens:

A. Tuberculosis
B. Typhus fever
C. Anthrax

Downloaded by AMAN SINGH (amansinghgkp8@gmail.com)


lOMoARcPSD|19252957

D.Dysentery
E. Leprosy

64. Constantly (in the body and in the external environment) pathogens form a
capsule:
A. Gas gangrene
B. Typhoid fever
C. Rhinoscleroma
D. Cholera
E. Diphtheria

65. Flagella of bacteria:

A. Participate in multiplication
B. Are antigens
C. cause bipolar coloration
D. Serve to preserve the species.
E. Consist of carbohydrates

66. The mobility of bacteria is determined by:

A. Polymerase reaction
B. In the hanging drop preparation
C.Luminescent microscopy
D. Coloring method by Indian ink
E.Gram staining

67. The motility of bacteria is provided by:

A. Reduced cell wall


B. Rotation of flagella
C. Fimbrias (pili)
D. enzyme activity
E. None of the above

68. Pili cause:

A. Motility

Downloaded by AMAN SINGH (amansinghgkp8@gmail.com)


lOMoARcPSD|19252957

B. Transformation
C. Conjugation
D. Transcription
E. Replication

69. Peritrich flagella are characteristic of pathogens:

A. Cholera
B. Tuberculosis
C. Typhoid fever
D. Dysentery
E. Diphtheria

70. Sporulation occurs under the following conditions:

A. Favorable external environment


B. Radiation
C. Internal body environment
D.High osmotic pressure
E. Stay in the soil

71. Microbial spores are important for:

A. Multiplication
B. Amplification
C. Preservation of species
D. Participation in the metabolism
E. Hybridization

72. Spores can form the following forms of microbes:

A. Spirils
B. All rod shape bacteria
C. Some types of rod-shaped
D. Cocci
E. Filamentary

73. Have the ability to spore-forming:

Downloaded by AMAN SINGH (amansinghgkp8@gmail.com)


lOMoARcPSD|19252957

A. The simplest
B. Bacteria
C. Klebsiella
D. Bacilli
E.Rickettsia

74. The high stability of the spore is due to:

A. The composition of the nucleoid


B. Structure and chemical composition of the shell
C. Increased potassium concentration
D. High free water
E. Glycogen content

75. Spores form pathogens:

A. Diphtheria
B. Typhus fever
C. Tetanus
D. Leprosy
E. Gonorrhea

76. The value of volutin granules:

A. Protection against adverse factors


B. Preservation of cell shape
C. Storage of nutrients
D. Participation in reproduction
E. Preservation of the species

77. Albert staining is used to identify:

A. Spores
B. Flagella
C. Nuclear substance
D. Capsules
E. Granules of volutin

78. Albert staining we use:

Downloaded by AMAN SINGH (amansinghgkp8@gmail.com)


lOMoARcPSD|19252957

A. Gentian violet solution


B. Methylene blue
C. Acetic acid solution of methylene blue
D. Fuchsin solution
E. Alcohol

79. Volutin granules are detected by coloring according to the method:

A. Gram
B. Ziel-Nielsen
C. By methylene blue
D. By Indian ink
E. Romanovsky - Giemsa

80. Preparation of the drug for microscopic examination includes:

A. Drying out a smear in a Pasteur oven.


B. Drying the smear in the flame.
C. Smear fixation in flame.
D. Filtration
E. Coloring bacteria without fixation.

81. Native non-stained preparations are prepared for microscopy:

A. Stereoscopic
B. Polarization
C. Fluorescent
D. Phasic Contrast
E. Electronic

82. When smear fixation occurs:

A. Activation of microbes.
B. Reduced optical density
C. Attaching to glass
D. Not susceptibility to the dye.
E. Detection of antigens.

Downloaded by AMAN SINGH (amansinghgkp8@gmail.com)


lOMoARcPSD|19252957

83. A simple staining method allows in a microbial cell:

A. Detect the shell


B. Determine the form
C. Detect nucleoid
D. Examine the ultrastructure
E. Identify antigens

84. The ability to receive dyes (tinctorial properties of bacteria) determine the
structure and composition:
A. Cytoplasm
B. Cytoplasmic membrane
C. Capsules
D. Plasmid
E. Cell wall

85. When coloring by the Gram method, apply ?

A. Carbol solution of gentian violet


B. Carbol solution of fuchsin
C. Vesuvine
D. Methylene blue
E. Acid treatment

86. Ziehl-Nielsen staining is used to identify:

A. Nuclear substance
B. Inclusions
C. Acid-Resistant Microbes
D. Motile microbes
E. Capsular microbes

87. When painting according to Ziehl-Nielsen, apply:

A. Acetic acid solution of methylene blue


B. Carbol solution of fuchsin
C. Solution of fuchsin
D. Iodine solution

Downloaded by AMAN SINGH (amansinghgkp8@gmail.com)


lOMoARcPSD|19252957

E. Ethyl alcohol

88. The acid resistance of microorganisms is associated with the presence of:

A. Nucleic acids
B. High salt concentrations
C. Polysaccharides
D. Fatty substances
E. Multilayer peptidoglycan

89. Acid-resistant bacteria include pathogens:

A. Pneumonia
B. Actinomycosis
C. Tuberculosis
D. Brucellosis
E. Leptospirosis

90. Gram stain depends on:

A. Shapes and sizes of cells


B. Forms of colonies
C. Features of the cytoplasmic membrane
D. Peptidoglycan content
E. Presence of flagella

91. Gram stained positively:

A. Bacilla
B. Escherichia
C.Gonococci
D. Rickettsia
E. Spirochetes

92. For morphology, spirochete is characterized by:

A. Rod-shaped
B. Elastic axial thread
C. Differentiated core

Downloaded by AMAN SINGH (amansinghgkp8@gmail.com)


lOMoARcPSD|19252957

D. Immobility
E. Sporulation

93. Each kind of spirochete is different:

A. The number and shape of curls


B. Intracellular parasitism
C. Type of food
D. Relation to Gram stain
E. Ability to lyse red blood cells

94. Spirochete morphology is studied:

A. In preparations "crushed" or "hanging" drop


B. In smears stained by Ziehl-Nielsen
C. Using a stereoscopic microscope
D. In Albert stained smears
E. Polarization microscopy

95. What does the term identification mean?


A. Determination of the type of microbes
B. Determination of antigenecity
B. Determination of pathogenecity
C. Determination of toxygenecity
D. Determination of microbe capsule

95. Which microorganisms use light and non organic substances as source of
energy and carbons?
A .Chemoautotrophs
B.Photoheterotrophes
C. Photoautotrophes
D. Chemoheterotrophes
E. Auxotrophes
96. Which microorganisms use foreign organic matter as a source of energy?
A. Heterotrophes

Downloaded by AMAN SINGH (amansinghgkp8@gmail.com)


lOMoARcPSD|19252957

B. Autotrophies
C.Chemotrophes
D.Auxotrophes
E.Prototrophes
97. Which microorganisms are capable of synthesis the organic compounds
they need?
A.Prototrophes
B.Heterotrophes
C.Auxotrophes
D. Autotrophies
E.Chemotrophes

98. What properties do saprophytes have?


A. Do not cause diseases
B. Always cause disease
C. Do not always cause disease
D. Do not cause disease in animals
E. Under certain conditions can cause disease
99. What culture medium us used for diagnostic purposes?
A. Liquid
B. Solid
C. Biological
D. Selective
E. Synthetic
100. What are the requirements for nutrient medium?
A. Sterility
B.Hypertonicity
C.Hypotonicity
D. Optimum temperature
E. Alkaline pH
101. What culture medium is differential diagnostic?
A. Meat-peptone agar
B. Blood agar

Downloaded by AMAN SINGH (amansinghgkp8@gmail.com)


lOMoARcPSD|19252957

C. Salt yolk agar


D. Meat peptone broth
E. Sugar broth
102. What is the reason for color changing of Salmonella typhi colonies on Mac
Conkey’s medium?
Lactose fermentation
Glucose fermentation
Maltose fermentation
Sucrose fermentation
Mannose fermentation
103. What does the term “pure culture” mean?
Embryo cell culture
Smear cell culture
Cell culture from other colonies
Cell culture from one colony
Cell culture from human tissue
104. By fermentation of which substances can bacteria be identified?
Proteins
Lipids
Vitamins
Nucleic acid
Minerals
105. What properties pigments have?
Participates in metabolism
B. Participates in reproduction
C. Participates in protection
D. Participates in respiration
E. Participates in spore formation
106. How you can character obligatory anaerobes?
A. Grow and multiply in presence of oxygen, so and without
B. They need free oxygen
C. They get energy by oxidation
D. They produce exotoxin

Downloaded by AMAN SINGH (amansinghgkp8@gmail.com)


lOMoARcPSD|19252957

E. They produce endotoxin


107. What type of microbes are parasites?
A. Heterotrophes
B.Prototrophes
C.Chemautotrophes
D.Phytotrophes
E. Saprophytes
108. Which culture medium is used for cultivation of anaerobes?
A. Mac Conkey
B. Robertson meat broth
C. Lowenstein- Jensen medium
D. EMB agar
E. Hiss medium
109. What device is used for cultivation of anaerobes?
A. Centrifugal machine
B. Oven
C. Anaerobic jar
D. Fridge
E.Thermostate
110. What method is used for isolation of pure culture of aerobic bacteria?
A. Heat method
B. Chemical method
C. Mechanical method
D. Physical method
E. Allergic method
111. Which microbes prefer 37º C for cultivation?
A.Psychrophiles
B. Mesophiles
C. Thermophiles
D. Halophiles
E.Aerophiles
112. Which factor help to bacteria for adhesion to the surface of the cell?

Downloaded by AMAN SINGH (amansinghgkp8@gmail.com)


lOMoARcPSD|19252957

A.Capsules
B.Flagella
C.Mezosomes
D.Pili
E.Spore

113.What microorganisms do mycoplasmas belong to?

A. Mushrooms
B.Protozoa
C.Viruses
D.Prions
E.Bacteria

114.What is characteristic of mycoplasmas?

A.Pleomorphism
B.Presence of capsule
C.Polimorphism
D.Intracellular parasitism
E.Gram positive coloration

115.What is characteristic of rickettsia?

A. Non cellular structure


B.Parasitism
C.The presence of flagella
D. Intracellular parasitism
E.Disjunctive reproduction

116.Which of the first works of Antony Levenguk do we know?

A. Put forward the theory of immunity


B. Discovered phagositosis
C. Suggested culture media
D. Constructed anautoclave
E. Saw and drew microbes

117. For what Ilya Ilyich Mechnikov received the Nobel prize?

A. Discovery of phagocytosis
B. Developed the principle of attenuation of microorganisms
C. Creator of the genetic engineering vaccine
D. Organizer of the medicine center

Downloaded by AMAN SINGH (amansinghgkp8@gmail.com)


lOMoARcPSD|19252957

E. Phenomenon of agglutination

118. What kind of work was famous for Louis Pasreur?

A. Designed first microscope


B. Discovered the causative agent of rabies
C. Proved that each type of fermentation has its own pathogen
D. He is founder of chemotherapy
E. He made a hepatitis vaccine

119. What merit in microbiology belongs to R.Koch?

A. Development of the allergic method


B. Suggested method of sterilization
C. Suggested aniline dyes and a condenser
D. Developed serological reactions
E. Suggested a method of determining antibiotics

120. Which method of sterilization is used for glassware?


A. Tyndalization
B. Steam fluid
C. Pasreurization
D. Autoclaving
E. Flaming

121. Which objects can be sterilized by ionizing radiation and ultrasound?


A. Fresh water
B. Tap water
C. Distilled water
D. Ground water
E. Mineral water

122. What method is effective for killing bacillus spores?


A. Drying
B. Pasreurization
C. Tyndalyzation
D. Autoclaving
E. Flaming

123. What does the term sterilization mean?


A. Destruction of pathogens for human microorganisms

Downloaded by AMAN SINGH (amansinghgkp8@gmail.com)


lOMoARcPSD|19252957

B. Desinfection of environmental objects


C. Partial destruction of microorganisms
D. Complete destruction of microorganisms
E. Prevention of microbes from entering the tissues of the human body

124.What definition can be given for antibiotics?

A. Highly active metabolic products of bacterial microorganisms


B. Substances that destroy cells of microorganisms
C. Substances that inhibit the reproduction of viruses
D . Substances with the some mechanism of action on microorganisms
E. Substances that do not cause the destruction of microorganisms

125. Which antibiotics inhibit the synthesis of the cell wall?

A. Polimixin
B. Streptomysin
C. Penicillin
D. Tetracycline
E. Erythromycin

126. Which antibiotic supresses the function of the cytoplasmic membrane of


microorganisms?
A. Macrolides
B. Polimuxins
C. Tetracycline
D. Penicillins
E. Streptomycin

127. Which antibiotics are obtained from Fungi?

A. Penicillins
B. Streptomycin
C. Tetracycline
D. Levomycetin
E. Erythromycin

128. For which microorganisms does the antibiogram determine antibiotic


sensitivity?

Downloaded by AMAN SINGH (amansinghgkp8@gmail.com)


lOMoARcPSD|19252957

A. Macroorganisms
B. Prions
C. Bacteria
D. Viroids
E. Bacteriophages

129. What are the common complications in patients with long antibiotic therapy?

A. Bacteriemia
B. Disbacteriosis
C. Sepsis
D. Septicipyomia
E. Virusemia

130. Antibiotics have a strong inhibitory effect on the growth and reproduction of
bacteria. What inhibition is caused by their mechanism of action?
A. Inhibition of the cytoplasmic membrane
B. Inhibition of the antigenic structure of bacteria
C. Inhibition of the synthesis of volutin
D. Inhibition of the function of the synthesis of spores
E. Inhibition of the biochemical activity of bacteria

131. Currently many types of bacteria are becoming resistant to antibiotics. As a


result of what mechanism do bacteria become resistant to antibiotics?
A. Changes in the morphology of bacteria
B. Acquisition of toxigenecity
C. Manifestations of pathogenicity enzymes
D. Changes in the bacterial chromosome due to mutation
E. Defective functioning of the microorganisms in immune system

132.Which microorganisms have the most pronounced antagonistic properties in


the human body?
A. Corynebacteria, mycobacteria
B. Mycoplasmas, chlamydia
C. Actinomycetes, fungi
D. Yersinia, francisella
E. Spirochetes, rickettsia

133. Which morphological property is characteristic of streptococci?

Downloaded by AMAN SINGH (amansinghgkp8@gmail.com)


lOMoARcPSD|19252957

A. The rod shape form of cells


B. The arrangement in the form of short chains
C. The presence of spores
D. The presence of flagella
E. The presence of volutin granules

134. What are the properties of streptococci?

A. Unculturing
B. Antigenic heterogenecity
C. The formation of endotoxin
D. The ability to ferment lactose
E. Resistance to penicillin

135. What method is used to identify streptococci by antigenic structure?

A. Infection of chicken embryo


B. Inoculation of mucus from the throat to the Mac Conkey medium
C. Inoculation of pathological material on the nutrient agar
D. Precipitation reaction
E. Agglutination reaction

136. What process is caused by mutations in microbe populations?

A. Integration of plasmids, transposons, is-sequences to cell chromosome


B. Transfer of genetic material from donor to recipient cell
C. Mistakes during conjugation
D. Reduced or increased concentration of oxygen
E. Depletion of nutrients in a cultural environment

137. What is the term of changing of DNA nucleotide sequence in bacteria cell?
A. Recombination
B. Mutation
C. Transformation
D. Conjugation

Downloaded by AMAN SINGH (amansinghgkp8@gmail.com)


lOMoARcPSD|19252957

E. Transcription

138. What does the term modification mean?


A. Changing of the primary DNA structure
B. Inability to adapt to the environment
C. Changing of cultural properties
D. Impossibility of reverse original type
E. Genotype changing

139. What mechanism of genetic recombination?

A. Transcription, translation, reproduction


B. Modification, replication, transduction
C. Conjugation, transformation, transduction
D. Mutation, dissociation, reactivation
E. Inversion, translation, transduction

140. What does the term transformation mean?

A. Transfer of genetic information by phages


B. Transition of RNA from cell to cell at contact
C. Inverted repeating of DNA sequences
D. Adaptive reaction in response to changing environment
E. Transfer of a DNA fragment from donor to a recipient.

141. What is the transfer of genetic material during transduction in bacteria cell?

A. Virulent phage
B. Temporary phage
C. Solution with DNA
D. Solution with RNA
E. Bacterial culture

Downloaded by AMAN SINGH (amansinghgkp8@gmail.com)


lOMoARcPSD|19252957

142. What stages does conjugation in bacteria consist of?

A. Connection donor cell F+ or Hfr with recipient cell


B. Change in DNA nucleotide sequences
C. Moving DNA fragments by replicon or between replicons
D. Integration of phage in to chromosome
E. Transfer of genetic material by phage

143. What related to extrachromosomal factors of heredity in bacteria?

A. Ribosomes
B. Lysosomes
C.Mesosomes
D. Plasmids
E.Volutin grains

144. Which information bacterial plasmids carry?


A. Basic for bacteria life
B. Help to survive in unfavorable conditions
C. Stimulates antibody production
D. Protect from phagocytosis
E. Participate in thermoregulation
Basic virology
145. Parasitism at the genetic level is characteristic of:
A. Bacteria
B. Rickettsia
C. Chlamydia
D. Mycoplasma
E. Viruses

146. To study the properties of viruses do not apply:


A. Microscopy
B. Cultivation
C. Animal Infection

Downloaded by AMAN SINGH (amansinghgkp8@gmail.com)


lOMoARcPSD|19252957

D. Ultracentrifugation
E. Gram Painting

145. Indication of viruses in laboratory animals:


A. Neutralization test
B. Formationof lesions
C. Characteristic clinic and the formation of intracellular inclusions
D. PCR
E. ELISA
146. Virus diagnostic method involves the identification of:
A.A virus antigen
B. Virus Nucleic Acid
C. Characteristic intracellular inclusions and elementary bodies
D. Hemadsorption
E. Formation of lesions

147. In order to maintain cell culture viability, the medium is used:


A. Endo
B. Lowenstein-Jensen
C. Blood agar
D. 199
E. Serum broth

148. Viruses are cultivated in:


A. Embryo
B. Simple Nutrient Media
C. Anaerobic conditions
D. Sugar Broth
E. 199 media

149. Reproduction of viruses includes:


A. Mitosis
B. Transcription
C. Binary division
D. Transformation
E. Conjugation

150. 740. Structural components of the virion:


A. Cytoplasmic membrane

Downloaded by AMAN SINGH (amansinghgkp8@gmail.com)


lOMoARcPSD|19252957

B. Mitochondria
C. Nucleic acid
D. Capsule
E. Ribosomes

151. Transfer of genetic information through free or naked DNA is called:


A. Transformation
B. Transduction
C. Conjugation
D. Lysogenic conversion
E. None of the above

152. The choice of material for the virological method depends on:
A. Type of nucleic acid
B. Type of symmetry capsomers
C. Accumulation of viral components
D. Antigenic structure
E. Clinics and pathogenesis of the disease

153. Cultivation of viruses:


A. Artificial nutrient media
B. Anaerobic conditions
C. Cell culture
D. 199 media
Е.Synthetic nutrient media

154. Isolation of the virus from clinical material by infecting a cell culture or
chicken embryo, followed by identification is a method:
A.Serologic
B.Biological
C. Virusoscopic
D.Virologists
E. Genetic

155. For cultivation does not apply:


A. Chicken Embryos
B. Organism of susceptible animals
C. Transformed cell cultures
D. Primary cell cultures

Downloaded by AMAN SINGH (amansinghgkp8@gmail.com)


lOMoARcPSD|19252957

E. 199 media

156. Virus identification in cell culture is carried out according to:


A. CPE
B. The nature of the colonies
C. Agglutination reactions
D. Hemagglutination inhibition reactions
E. Biochemical reactions.

157. Identification growth of the virus in cell culture allows the presence of:
A. Characteristic colonies
B. Specific antibodies
C. Pathological changes in cells
D.Protolithic enzymes
E. Toxins

158. Visual detection of a virus or intracellular inclusions directly in the test


material is a method:
A. Virological
B.Biological
C. Virus-scopic
D. Molecular genetic
E. Serological

159. Bacteriophage multiplication occurs in:


A. Chicken embryos
B. Cells of any bacterial cultures
C. Animal Body
D. Certain species of bacterial cells
E. Artificial nutrient media

160. Indications of viruses during cultivation on chicken embryos are carried


out on the basis of:
A. Neutralization test
B. Blast transformation
C. Cytopathic effect
D. The nature of specific lesions of the membranes and body of the
embryo
E. Agglutination reactions

Downloaded by AMAN SINGH (amansinghgkp8@gmail.com)


lOMoARcPSD|19252957

161. Bacteriophages are characterized by:


A. Cellular structure
B. The content of nucleic acids-DNA and RNA
C. Content of one nucleic acid is DNA or RNA
D. Gram Positive staining
E. High prevalence in the air

162. The size of the phage set:


A. Eyepiece - micrometer
B. Fluorescent microscopy
C. Phase contrast microscopy
D. Ultracentrifugation
E. Stereoscopic microscopy

163. Virulent phages are characterized by:


A.Symbiosis with a bacterial cell
B. Lysis of the bacterial cell
C. Biosynthesis of phage components in the medium 199
D. The presence of pathogenicity enzymes
E. Synchronous replication with the bacterial cell gene

164. For a prophages it is characteristic:


A. Excluded from the chromosome of the cell and become virulent
B. Embed your DNA in the chromosome of plants
C. To autonomous reproduction in a bacterial cell and its lysis
D. Change the properties of plants
E. Conjugate

165. Bacteriophages (everything is correct, except):


A. Are of synthetic origin
B. Bacteria viruses their natural enemies
C. Antibacterial drugs
D. Unlike antibiotics, human cells are not affected.
E. Bacteriophage preparations do not have side effects (can be prescribed
to newborns, pregnant and lactating)

Downloaded by AMAN SINGH (amansinghgkp8@gmail.com)


lOMoARcPSD|19252957

166. Not used bacteriophages for:


A. Treatments
B. Differentiation of bacteria
C. Prevention
D. Determination of antibiotic resistance
E. Indications of bacteria

167. The phenomenon of bacteriophage has been studied in detail:


A.Ivanovsky
B. Koch
C. Metchnikov
D. d'Herell
E. Pasteur

168. A productive type of viral reproduction includes:


A. Biosynthesis of viral components in the cell
B.Lysis of viruses
С. Embedding virus nucleic acid in the chromosome of the cell
D. Changing the properties of the cell
E. Synchronous replication of the viral genome with the cellular genome

169. The integrative type of interaction (virogenation) includes:


A. Cytopathic effect of the virus
B. Biosynthesis of viral components in the cell
C. Embedding the nucleic acid of the virus into the chromosome of the
cell
D. Virus exit from the cell
E. Cell death

170. Result productive interaction of the virus with the cell:


A. Antigenic cell transformation
B. Persistence of the virus
C. Virogenation
D. Oncogenic cell transformation
E. Cell death

171. Which of the following characteristics are true for viruses?


A. Obligate intracellular infective agents
B. Contain either DNA or RNA

Downloaded by AMAN SINGH (amansinghgkp8@gmail.com)


lOMoARcPSD|19252957

C. Do not multiply by binary fission


D. Cannot grow in artificial media
E. All of the above

172. The main properties of viruses:


A. Contain the same type of nucleic acid (DNA or RNA)
B. Contain both types of nucleic acids.
C. Capable of growth and binary division
D. Have their own protein synthesis systems
E. Have their own energy-efficient systems

173. Microscopic cytopathic effect of viruses in the culture of labels is


manifested in:
A. Conserving morphology of cells
B. Formation of giant multicore labels (symplasts)
C. Change in antigenic structure
D. Conservation of nuclei
E. Color Change

174. Which type of interaction between virus and host cell is called as lysogeny?

A. Productive
B. Integrative
C. Abortive
D. Virulent
E. Defective
175. What kind of laboratory tests can be done by using bacteriophage?
A. Identification of microbe
B. Study of internal structures
C. Detection of antibiotic sensitivity
D. Detection of serotype
E. Biochemical identification
176. In which case bacteriophage is not used?
A. Treatment

Downloaded by AMAN SINGH (amansinghgkp8@gmail.com)


lOMoARcPSD|19252957

B. Identification
C.Profilaxys
D. Detection of antibiotic resistance
E. Indication
178. Which process is caused by productive type reproduction of virus in host
cell?
A. Biosynthesis of viral components
B.Lysis of viral particles
C. Integration of viral nucleic acid to chromosome
D. Changing properties of the cell
E. Replication of viral genome with chromosome
179. Which process is caused by integrative type reproduction of virus in host cell?
A.Cytopathic action of virus
B. Biosynthesis of the viral components
C. Integration of viral nucleic acid
D. Virus release from the host cell
E. Death of the cell
180. What changes is caused by virulent interaction between virus and host cell?
A. Antigenic transformation
B. Virus persistence
C.Lysogeny
D. Oncogenic transformation
E. Cytopathyc action
181. What function have viral nucleic acid after entering the host cell?
A. Involved in cell division
B. Carries new genetic information
C. Does not affect the cell internal structures
D. Activates cell metabolism
E. Causes the cell to produce pathogenic enzymes
182. What are the main properties of viruses?
A. Breathable
B. Capable to protein synthesis
C. Capable for growth and binary fission

Downloaded by AMAN SINGH (amansinghgkp8@gmail.com)


lOMoARcPSD|19252957

D. Capable for spore production


E. Capable to reproduction
183. What effect of viruses is called as cytopathyc?
A. Changing cell antigens
B. Symplasts
C. Core conservation
D. Production of volutin grains
E. Discoloration
184. At what level do viruses intracellularely parasitize?
A. Genetic
B. Cytoplasmic
C. Energy
D. Ribosomal
E. Cellular

185. Which microorganisms are genetic parasites?

A. Actinomysetes
B. Rickettsy
C. Chlamydia
D. Mycopasma
E. Viruses

186. What factor play role in interaction between virus and host cell?

A. Form of the viruses


B. Correspondence of the receptors
C. Type of reproduction
D. Correspondence of the viral envelope
E. Type of nucleic acid

187. What method is used to study the cytopathic effect of viruses?

A. Cultivation on cell culture


B. Cultivation on culture media
C. Cultivation under anaerobic conditions
D. Cultivation on complex culture media
E. Cultivation on selective media

188. Who is the founder of virology?

Downloaded by AMAN SINGH (amansinghgkp8@gmail.com)


lOMoARcPSD|19252957

A. R.Koch
B. L. Pasreur
C. Z.Vinogradsky
D. D.Ivanovsky
E. D.Zabolotnyi

189. Which test can determine a viral infection?

A. Determination of the presence of the virus


B. Determination of the antigen of the virus
C. Determination of the type of virus
D. Determination of the genotype of the virus
E. Determination of the pathogenicity of the virus

190. What can be studied with a viroscopic method of diagnostic?

A. Virus antigens
B. Virus nucleic acid
C. Typical intracellular inclusions
D. Virus receptors
E. Virus symmetry type

191. Which culture medium is used to maintain cell culture viability?

A. Mac Conkey`s agar


B. Jewenstein-Jensens medium
C. Blood agar
D. 199 media
E. Serum broth

192. Which culture medium is used for virus cultivation?

A. Shell of the chicken embryo


B. Simple culture media
C. Blood agar
D. Sugar broth
E. Enriched media

193. Which proses is one of the stages of virus reproduction ?

A. Mutation
B. Transcription
C. Transformation
D. Transduction

Downloaded by AMAN SINGH (amansinghgkp8@gmail.com)


lOMoARcPSD|19252957

E. Conjugation

194. Which structure is the main component of the virion?

A. Cytoplasmic membrane
B. Mytochondria
C. Nucleic acid
D. Capsule
E. Ribosomes

195. What does the concept virogenesis the interaction of the virus with the host
cell mean?

A. The stage of reproduction of the virus


B. Integration of the viral nucleic acid
C. Method of reproduction
D. Mechanism of death of the virus
E. Method of protection

196. What kind of cell cultures can be used and less for virus cultivation?

A. Chicken embryo cell culture


B. Secondary cell culture
C. Transplantable cell cultures
D. Primary cell culture
E. Diploid cell culture

197. What are the manifestations of determining the indication of viruses?


A. Hem agglutination reaction
B. Coagglutination reaction
C. Agglutination reaction
D. Precipitation reaction
E. Biochemical reaction

198. Which manifestations indicate the growth of viruses in cell culture?


A. R-type of colonies
B.S-type of colonies
C. Intra cellular inclusions
D. Proteolitic enzymes
E. Manifestation of hemolysis

199. Where does the bacteriophage can multiply?

Downloaded by AMAN SINGH (amansinghgkp8@gmail.com)


lOMoARcPSD|19252957

A. 12 day chicken embryo


B. Cells of any bacterial cultures
C. The organism of laboratory animals
D. Bacterial cell of a certain type
E. Artificial culture media

200. How does virus can be identified during cultivation on the chicken embryo?
A. Colored sample test
B. Blast transformation
C. Cytopathic its effect
D. Coagglutination reaction
E. Agglutination reaction

201. What are the morphological features of bacteriophages?

A. Cell structure
B. Presence of DNA and RNA
C. Absence of DNA and RNA
D. Non specificity
E. Prevalence in air

202. What are the properties of virulent phages?


A. Symbiosis with a bacterial cell
B. Lysis of the bacterial cell
C. Persistence in the bacterial cell
D. The presence of pathogenicity enzymes
E. Synchronous replication with the bacterial cell gene

203. Which properties do bacteriophages possess?


A. Exclude from the chromosome of the cell and become virulent
B. Added DNA in the chromosome of the plant
C. To autonomous reproduction in the bacterial cell and its lysis
D. Change the properties of the cell
E. Conjugate

Basic immunology
204. What is the main drug for treatment of infection caused by exotoxin?
A. Interferon

Downloaded by AMAN SINGH (amansinghgkp8@gmail.com)


lOMoARcPSD|19252957

B. Interferon inductors
C. Immune serum
D. Phages
E. Precipitate serum
205. Which of the following statements is true about hapten?

A. It induces brisk immune response


B. It needs carrier to induce immune response
C. It is a T-independent antigen
D. It has no association with MHC
E. It is a B-independent antigen

206. Which of the following best denotes classical complement pathway activation
in immuno- inflammatory condition?

A. C2, C4 and C3 decreased


B. C2 and C4 normal, C3 is decreased
C. C3 normal and C2 C4 decreased
D. C2, C4, C3 all are elevated
E. C4, C5 are elevated

207. Which is not a macrophage?

A. Monocyte
B. Microgila
C. Kupffer cells
D. Lymphocytes
E. PMNL

208. Common between B and T cells:

A. Origin from same cell lineage


B. Site differentiation
C. Antigenic marker
D. Both humoral and cellular immunity
E. Further differentiation is not seen

Downloaded by AMAN SINGH (amansinghgkp8@gmail.com)


lOMoARcPSD|19252957

209. All of these are antigens presenting cells except:

A. B cells
B. Fibroblasts
C. Langerhans cells
D. None of the above
E. All

210. Perforins are produced by:

A. Cytotoxic T cells
B. Suppressor T cells
C. Memory helper T cells
D. Plasma cells
E. NK cells

211. Active immunity can be induced by:

A. Toxoids
B. Antibiotics
C. Antitoxin
D. Immunoglobulins
E. Maternal antibody

212. True about passive immunity:

A. Can not be given with active immunity


B. Last for 405 days only
C. It can be given before disease occurrence
D. Toxoid
E. Takes long time to develop

213. Interferon:

A. Is not species specific


B. Reacts directly with virus particles to inactivate them
C. Reacts with cells and the affected cell then becomes resistant to a number of
different viruses
D. Constitutively produced at high levels in cells but requires an inducer for activity

Downloaded by AMAN SINGH (amansinghgkp8@gmail.com)


lOMoARcPSD|19252957

E. Reacts with bacterial cells

214. Phagocytic cells elaborate:

A. Prostaglandins
B. Thromboxane
C. Interferons
D. All of the above
E. None of the above

215. CD4 are associated with:

A. Helper T cell
B. Sypressor T cells
C. NK cells
D. T cells antigen receptor complex
E. All of the above

216. Vaccination is based on the principle of:

A. Agglutination
B. Phagocytosis
C. Immunological memory
D. Clonal detection
E. Opsonization

217. The reaction between antibody and soluble antigen is demonstrated by:

A. Agglutination
B. Precipitation
C. Complement fixation
D. Hemagglutination test
E. ELIZA

218. A single immunoglobulin molecule contains:

A. 1 light chain, 1 heavy chain


B. 2 heavy chains, 1 light chain
C. 2 light chains, 2 heavy chain

Downloaded by AMAN SINGH (amansinghgkp8@gmail.com)


lOMoARcPSD|19252957

D. 2 light chains, 1 heavy chain


E. 1 light chain, 2 heavy chains

219. Antigen binding site on antibody is:

A. Hinge region
B. Constant region
C. Variable region
D. Hypervariable region
E. All of the above

220. IgE is secreted by:


A. Mast cell
B. Eosinophils
C. Neutrophils
D. Basophils
E. Plasma cells

221. Classic complement is activated by:


A. IgG
B. IgA
C. IgE
D. IgD
E. None of the above

222. Which of the following immunoglobulins can cross placenta?


A. IgA
B. IgM
C. IgG
D. IgD
E. IgE

223. True about antibody:


A. IgM is produced in primary response
B. IgD protects mucosa
C. IgE is main antibody in secondary response
D. IgG is not main antibody in secondary' response
E. IgA is for secondary response

Downloaded by AMAN SINGH (amansinghgkp8@gmail.com)


lOMoARcPSD|19252957

224. Antibodies present in person with O blood group:


A. Anti-A antibody only
B. Anti-B antibody only
C. Both Anti-A and Anti-B antibody
D. No antibody
E. Anti-O antibody

225. Regarding IgE which of the following is false.


A. Cause anaphylaxis
B. Immediate reaction
C. Fix complement
D. Cross placenta
E. None of the above

226. The exact part of the antigen that reacts with the immune system is called as:

A. Clone
B. Epitope
C. Idiotope
D. Effector
E. Hinge region

227. Function of IgA is:


A. Acts as a mucosa barrier for infection
B. Circulating antibody
C. Kills virus infected cells
D. Activates macrophages
E. Causes delayed hyper sensitivity reaction

228. The following constitutes approximately 75% of total immunoglobin in


humans:
A. IgG
B. IgM
C. IgE
D. IgA
E. IgD

229. The immunoglobulin that can cross the placenta:

Downloaded by AMAN SINGH (amansinghgkp8@gmail.com)


lOMoARcPSD|19252957

A. IgG
B. IgM
C. IgE
D. IgA
E. IgD

230. Structure of antibodies is composed of:


A. Single peptide chain
B. Two peptide chains
C. Non sulphur amino acid
D. 2 long and 2 short peptide chain
E. None of the above

231. Opsonization occurs due to:


A. Endotoxin
B. Complement
C. IgM
D. IgG
E. IgA

232. First antibody response is mediated by:


A. IgD
B. IgM
C. IgA
D. IgE
E. Ig G

233. Antibody transfer mother to fetus is:


A. IgG
B. IgM
C. IgD
D. IgA
E. None

234. Atopy is mediated by:


A. IgE
B. IgD
C. IgM
D. IgA

Downloaded by AMAN SINGH (amansinghgkp8@gmail.com)


lOMoARcPSD|19252957

E. Ig G

235. Immunoglobin are involved in type 1 hypersensitivity reaction is:


A. IgE
B. IgD
C. IgM
D. IgA
E. Ig G

236. Coombs test is:


A. Precipitation test
B. Agglutination test
C. CFT
D. Neutrilization test
E. ELIZA

237. Which of the following is an example of type IV hypersensitivity?


A. Arthus reaction
B. Serum sickness
C. Schwartzman reaction
D. Granulomatous reaction
E. Anaphylaxis

238. Which of the following is false?


A. Theobald-Smith phenomenon is a type I hyper-sensitivity reaction
B. Serum sickness is a type II hypersensitivity reaction
C. Allograft rejection is a type IV hypersensitivity reaction
D. Transfusion reaction is a type II hypersensitivity reaction
E. None of the above

239. Hemolytic disease of newborn is which type of hypersensitivity reaction?


A. Type -1
B. Type - II
C. Type - III
D. Type-IV
E. Type - V

240. Skin test is used for which hypersensitivity reaction except:


A. I

Downloaded by AMAN SINGH (amansinghgkp8@gmail.com)


lOMoARcPSD|19252957

B. II
C. III
D. IV
E. All

241. Skin test based on neutralization reaction is/are:


A. Casoni test
B. Lepromin test
C. Tuberculin test
D. Schick test
E. None of the above

242. Type-1 hypersensitivity includes all of the following except:


A. Autoimmune hemolytic anemia
B. Anaphylaxis
C. Extrinsic asthma
D. Hay fever
E. Atopy

243. True about anaphylaxis:


A. Type-1 reaction
B. Large amount of histamine released
C. Cytokines like IL4, IL5, and IL6 and GMCSF are released
D. Mediated through allergen specific IgE
E. All of the above

244. All the following types of hypersensitivity reactions can be demonstrated


by skin test except:
A. Type I
B. Type II
C. Type III
D. Type IV
E. None of the above

245. Delayed hypersensitivity reaction is mediated by the following:


A. B lymphocytes
B. NK cells
C. Mast cells
D. T lymphocytes

Downloaded by AMAN SINGH (amansinghgkp8@gmail.com)


lOMoARcPSD|19252957

E. None of the above

246. Primary immune response is mediated by:


A. IgE
B. IgM
C. IgA
D. IgD
E. IgG

247. All are type-II hypersensitivity reaction except:


A. Hemorrhagic disease of newborn
B. Grave's disease
C. Autoimmune diseases
D. Hemolytic anemia
E. None of the above

248. Type I hypersensitivity, the mediator is:


A. IgE
B. Ig E
C. IgM
D. IgC
E. Ig D

249. Erythroblastosis fetalis is an example of which type of hypersensitivity


reaction?
A. Type I
B. Type II
C. Type III
D. Type IV
E. None of the above

250. Soil microorganisms are involved in the processes:


A. Biological fixation of complement
B. Ammonification
C. Hemolysis
D. Bacteriolysis
E. Formation of drug resistance

251. Soil is the natural habitat for pathogens:

Downloaded by AMAN SINGH (amansinghgkp8@gmail.com)


lOMoARcPSD|19252957

A. Deep mycoses
B. Diphtheria
C. Tuberculosis
D. Gonorrhea
E. Flu

252. In the soil, the pathogens retain their viability for years:
A. Anthrax
B. Tuberculosis
C. Leptospirosis
D. Q fever
E. Brucellosis

253. Soil may serve as a transmission factor:


A. Deep mycoses
B. Syphilis
C. Hepatitis B
D. Gonorrhea
E. HIV infection

254. By water spread:


A. Gonorrhea
B. Cholera
C. Syphilis
D. Typhus fever
E. Relapsing fever

255. Airborne spread:


A. Typhus fever
B. Typhoid fever
C. Plague
D. Rabies
E. Gonorrhea

256. Disease transferred by blood


A. Hepatitis B
B. Typhoid fever
C. Hepatitis A
D. Rabies

Downloaded by AMAN SINGH (amansinghgkp8@gmail.com)


lOMoARcPSD|19252957

E. Gonorrhea

257. Clinical and subclinical infections lead to:


A. Active natural immunity
B. Passive natural immunity
C. Active artificial immunity
D. Passive artificial immunity
E. All of the above

258. Obligatory and representatives of the microflora of the colon are:


A. Vibrioes
B. Streptococcus
C. Bacteroids
D. Staphylococcus
E. Shigella.

259. On a well-formed intestinal microbiocenosis is indicated by the


predominance of:
A. Paratyphoid bacrteria
B. Shigella
C. Bifidumbacterium
D.Enterococci
E. Staphylococcus

260. Microbiological manifestations of colon dysbacteriosis:


A. Absence of bifidum bacteria
B. Increasing the amount of lactobacilli
C. The presence of E. coli
D. Shigella appearance
E. Presence of enterococci

261. Preparations normalizing the microflora of the large intestine:


A. Antibiotics
B. Sulfonamides
C. Anatoxin
D. Bifidumbacterin
E. Bacteriophages

262. The most contaminated digestive tract:

Downloaded by AMAN SINGH (amansinghgkp8@gmail.com)


lOMoARcPSD|19252957

A. Oral cavity
B. Esophagus
C. Stomach
D. Small intestine
E. Gallbladder

263. Which Ig will find in acute infection?


A. IgA
B. IgM
C. IgD
D. IgE
E. All of the above

264. Classical pathway of the complement is activated by:


A. Antigen
B. Antibody
C. Antigen –antibody complex
D. Protein
E. None of the above

265. The term "infection" is derived from the Latin word infectio, which means:
A. Carriage of bacteria
B. Freedom from anything
C. Infection
D. Persistence
E. Return of symptoms

266. Infectious diseases the source of which is the only human called:
A.Sapronosis
B. Zoonosis
C. Anthroponoses
D. Bacteriosis
E. Zooanthroponosis

267. Infectious morbidity is called sporadic with:


A. Single cases of a specific infection in a particular region
B. Coverage of a specific infection of the entire planet.
C. Involving a specific infection in several regions
D. A sharp excess in a certain period of the level of sporadic infection

Downloaded by AMAN SINGH (amansinghgkp8@gmail.com)


lOMoARcPSD|19252957

E. None of the above

268. Infectious disease is:


A. Prolonged presence of viruses in humans
B. Complex process of interaction of pathogenic microorganisms
C. Interaction between host and microorganism
D. Cohabitation, beneficial for micro - and microorganisms
E.Microbial life due to the microorganism

269. Invasiveness - the ability of microbes:


A. Suppress the body's defenses
B. Penetrate the body tissue
C. Cause an infectious process
D. Attach to cell surface
E. To multiply inside the cells

270. Pathogenicity is the potential capacity of microbes:


A. Forming immunity
B. Lysed with phages
C. Ferment carbohydrates
D. Cause infection
E. Cleave proteins

271. Virulence is a microbe property that determines the degree of:


A. Enzyme activity
B. Immunogenicity
C. Pathogenicity
D. Resistance
E. Variability

272. The minimum lethal dose in microbes is measured:


A. Pathogenicity
B. Virulence
C. Antigenicity
D. Enzymatic activity
E. Phagolysis

273. The number of bacteria capable of causing an infectious disease is


measured by the dose:

Downloaded by AMAN SINGH (amansinghgkp8@gmail.com)


lOMoARcPSD|19252957

A. Minimal lethal (Dlm)


B. Definitely deadly (Dcl),
C. Infectious (ID)
D. Killing half of infected animals (LD50)
E. Unit of Action (ED)

274. For exotoxins it is characteristic:


A. Lipopolysaccharide nature
B. High specificity
C. Stability
D. Low toxicity
E. Biochemical activity

275. The nature of exotoxins:


A. Lipopolysaccharide secreted into the environment
B. Lipopolysaccharide, associated with the body of the microbial cell
C. Protein, low toxic and non specific
D. Protein, not released into the environment
E. Protein, highly toxic, secreted into the environment.

276. Toxoid are obtained from:


A. Antitoxic sera
B. Allergens
C. Exotoxins
D. Endotoxins
E. Agglutinating sera

277. Antitoxic serum does not apply to the treatment of:


A.Tetanus
B. Diphtheria
C. Gas gangrene
D. Botulism
E. Syphilis

278. Microbial endotoxins are characterized by:


A. Lipopolysaccharide nature
B. High specificity
C. Protein nature
D.Lability

Downloaded by AMAN SINGH (amansinghgkp8@gmail.com)


lOMoARcPSD|19252957

E. Strong toxicity

279. Bacteria Endotoxins:


A. Formed in the process of life
B. Very poisonous
C. Strong antigens
D. Released upon microbial cell death
E. Transfer to toxoid

280. Toxoid are characterized by


A. Specificity
B. Polysaccharide nature
C. Toxicity
D. Necrotic action
E. Antibiotic Sensitivity

281. Toxoid are used to:


A. Preparation of antibacterial sera
B. Conduct desensitization
C. Formulations of skin allergy tests
D. Creation of antitoxic immunity
E. Preparation of Agglutinating Sera

282. The form of symbiosis, in which the microbe causes obvious harm to its
host, is called:
A. Commensalism
B. Metabiosis
C. Parasitism
D. Mutualism
E. Antagonism

283. The form of symbiosis, in which both partners gain mutual benefit, is
called: A. Commensalism
B. Metabiosis
C. Parasitism
D. Mutualism
E. Antagonism

Downloaded by AMAN SINGH (amansinghgkp8@gmail.com)


lOMoARcPSD|19252957

284. Relationships when a microorganism uses a microorganism without harm,


but without obvious benefit, are called:
A. Mutualism
B. Commensalism
C. Parasitism
D. Satellite
E. Antagonism

285. The ability of microbes to stick on the surface of sensitive cells is called:
A. Phagocytosis
B. Pathogenicity
C. Adhesiveness
D. Toxigenicity
E. Virulence

286. The time period from the moment of infection to the onset of the first
symptoms of the disease is called:
A. Recovery
B. Incubation
C. Prodromal
D. The height of the disease
E. None of the above

287. The appearance of the first symptoms of the disease is associated with the
period:
A. Incubation
B. Prodromal
C. The height of the disease
D. Recovery
E. Bacterial carriers

288. The existence of an infectious disease in any particular locality is


designated by the term:
A. Epidemic
B. Endemic
C. Pandemic
D. Bacteremia
E. Septicemia

Downloaded by AMAN SINGH (amansinghgkp8@gmail.com)


lOMoARcPSD|19252957

289. Isolation of the pathogen, continuing after recovery, is called:


A. Mixed infection
B. Autoinfection
C. Sepsis
D. Bacterial carrier
E. Bacteremia

290. Repeated infection of the body by the same pathogen after recovery is
called: A. Mixed infection
B. Bacteremia
C. Reinfection
D. Septicopyemia
E. Septicemia

291. Repeated infection with the same type of microbe until recovery is called:
A. Recurrence
B. Bacterial carrier
C.Superinfection
D. Reconvalescence
E. Reinfection

292. The return of the clinical manifestations of the disease without re-infection
due to pathogens remaining in the body is called:
A. Mixed infection
B. Monoinfection
C. Reinfection
D. Recurrence
E .Superinfection

293. The period of an infectious disease:


A. Incubation
B. Prodromal
C. Symptoms
D. Reconvalescence
E. All of the above

294. The characteristics of endogenous infections:


A. Change of clinical periods
B. Cyclicity

Downloaded by AMAN SINGH (amansinghgkp8@gmail.com)


lOMoARcPSD|19252957

C. Duration of the incubation period


D. No incubation period
E. Penetration of the pathogen from the external environment into the body

295. The outbreak of an infectious disease among animals is called:


A. Pandemic
B. Epidemic
C. Endemic
D. Epizootia
E. Zoonosis

296. The rapid spread of infectious diseases across countries and continents is
called:
A. Epidemic
B. Epizootic
C. Pandemic
D. Endemic
E. Sporadic morbidity

297. Mass infectious diseases spreading across countries and continents are
called by the term:
A. Epidemic
B. Endemic
C. Pandemic
D.Integration
E.Penetration

298. Infections that develop after the penetration of bacteria into the human
body from environmental objects (soil) are called:
A. Epizootia
B. Endemic
C. Pandemic
D. Epidemic
E. Sapronosis

299. Diseases caused by medical intervention are called:


A. Endogenous
B. Autoinfection
C. Iatrogenic

Downloaded by AMAN SINGH (amansinghgkp8@gmail.com)


lOMoARcPSD|19252957

D. Exogenous
E. Manifest

300. Nosocomial infections are otherwise called:


A. Autoinfection
B. Endogenous
C. Inaparat
D. Exogenous
E. Hospital

301. Mechanism of transmission of the pathogen through blood-sucking


arthropods:
A. Fecal-oral
B. Airborne
C .Contact
D. Transmissible
E. Transplant

302. The vertical mechanism of transmission is the transmission of a


microorganism:
A. Through public goods
B. By direct contact
C. From mother to fetus
D. Fecal-oral
E. Airborne

303. Ways of transmission of the pathogen in the fecal-oral mechanism through:


A. Air
B. Flea
C. Water
D. Mosquitoes
E. Blood

304. At endogenous infection, microorganisms enter the body with:


A. Water
B. By air
C. soil
D. Food
E. None of the above.

Downloaded by AMAN SINGH (amansinghgkp8@gmail.com)


lOMoARcPSD|19252957

305. The spread of microorganisms from the primary site by the hematogenous
route is called:
A. Sepsis
B. Bacteremia
C. Septicopyemia
D. Septicemia
E. Toxinemia

306. Microbes do not spread throughout the body by:


A. Hematogenous
B. Lymphogenic
C. Urogenic
D. Neurogenic
E.Transmissive

307. When microbes multiply in the blood, the following occurs:


A. Bacteremia
B. Viremia
C. Toxinemia
D. Septicopyemia
E. Septicemia

308. In the multiplication of microbes in the blood and the simultaneous


formation of purulent foci in the internal organs, the following occurs:
A. Bacteremia
B. Septicopyemia
C. Toxinemia
D. Viremia
E. Septicemia

309. The first barrier to the penetration of microbes into the internal
environment of the body is:
A. Subcutaneous tissue
B. Lymph nodes
C. Phagocytosis
D. Skin and mucous membranes
E. Inflammation

Downloaded by AMAN SINGH (amansinghgkp8@gmail.com)


lOMoARcPSD|19252957

310. The entry of microbes into the blood, their reproduction and realizing toxin
is called the term:
A.Bacteremia
B. Sepsis (septicimia)
C. Toxinemia
D. Septicopyemia
E. Endemia

311. Returning the symptoms of the same disease without re-infection is called:
A. Bacteremia
B. Reinfection
C. Superinfection
D. Latent
E. Septicemia

312. The introduction of antitoxic serum is called:


A. Serotherapy
B.Surgery
C. Chemotherapy
D.Seroindication
E. All of the above

313. Infectiousness of viruses depends on:


A. Envelope and capsid
B. DNA and RNA
C. Hemagglutinin and Neuraminidase
D. Enzymes
E. None of the above

314. The prolonged presence of viruses in the human body is called:


A. Viresemia
B. Transduction
C. Lysogenesis
D. Phage conversion
E. Persistence

315. After an illness, the body forms an immunity:


A. Natural

Downloaded by AMAN SINGH (amansinghgkp8@gmail.com)


lOMoARcPSD|19252957

B. Actively Acquired
C. Passively Acquired
D. Nonspecific
E. Artificially active

316. Introduction of toxoid forms the immune system in it:


A. Infectious, non-sterile.
B. Natural active.
C. Artificial active.
D. Artificial passive
E. Natural passive

317. Virulence of microbes is caused by:


A. Oxidation enzymes
B. Biochemical activity
C. Sporulation
D. Toxin formation
E. Resistance

318. The ability of microorganisms for adhesion and colonization is a


manifestation of:
A. Variability
B. Immunogenicity
C. Virulence
D. Avirulence
E. Aggression

319. To study the hemolytic capacity of microbes, the medium is used:


A. MacConkey
B. Nutrient agar
C. Serum agar
D. Blood agar
E. Charcoal agar

320. Agar is used to study the lecithinase activity of microbes:


A. Blood
B. Alkaline
C. Yolk-Salt
D. MPA

Downloaded by AMAN SINGH (amansinghgkp8@gmail.com)


lOMoARcPSD|19252957

E. Charcoal

321. Enzymes: hyanuronidase, neurominidase are:


A. Remedial
B. Proteolytic
C. Hydrolytic
D. Oxidizing
E. Pathogenicity

322. Enzymes: lecithinase, DNase, phosphatase are:


A. Oxidative
B. Pathogenicity
C. Restorative
D. Hydrolytic
E. Fermentation

323. Microorganisms produce exotoxins:


A. During life
B. Periodically
C. After the death
D. Formalin treatment
E. In no case

324. The complement system, lysozyme, properdin, and leukyns belong to


protection factors:
A. Humoral specific
B. Cellular specific
C. Humoral nonspecific
D. Cell nonspecific
E. None of the above

325. Macrophages take part in:


A. Recombinations
B. Differentiation of antigens
C. Immediate-type hypersensitivity
D. Cooperation T and B lymphocytes
E. Preservation of immunological memory

326. Antibody opsonin function:

Downloaded by AMAN SINGH (amansinghgkp8@gmail.com)


lOMoARcPSD|19252957

A. Lyse the red blood cells


B. Activate phagocytosis
C. Agglutinate bacteria
D. Suppress phagocytosis
E. Neutralizing viruses

327. The central organs of the immune system are:


A. Blood and lymph
B. Lymph nodes
C. Liver and spleen
D. Thymus and bone marrow
E. Brain and cerebellum

328. Peripheral organs of the immune system include:


A. Thymus
B. Lung
C. Lymph nodes
D. Bone marrow
E. Pancreas

329. The ancestor of all immune cells is:


A. Macrophage
B. Stem cell
C. Erythrocyte
D. Platelet
E. White blood cell

330. Antitumor, antiviral, transplant immunity is performed by:


A. Macrophages
B. B lymphocytes
C. Dendritic cells
D. T-killers
E. Plasma cells

331. The function of plasma cells is:


A. Interferon products
B. Interleukin production
C. Synthesis of immunoglobulins
D. Timosin production

Downloaded by AMAN SINGH (amansinghgkp8@gmail.com)


lOMoARcPSD|19252957

E. Graft rejection

332. Reaction of phagocytosis begins with:


A. Agglutination of microbes
B. Lysis of microbes
C. Microbial adsorption
D. Immunoglobulin synthesis
E. Positive chemotaxis

333. Interferons, interferon inductors are drugs against:


A.Bacteria
B. Fungi
C. The Simplest
D. Viruses
E. Toxins

334. Interferon stimulates cell synthesis of a protein kinase protein that acts on:
A. Prions
B. Viruses
C. Bacteria
D. Rickettsia
E. Chlamydia

335. The main antiviral humoral factor of nonspecific immunity is:


A. Interferon
B. Lysozyme
C.β-lysines
D. Immunoglobulins
E. Arbidol

336. Lysozyme:
A. Agglutinates bacteriophages
B. Neutralizes viruses
C. Lyses Erythrocytes
D. Lyses bacteria
E. Activates phagocytosis

337. Immune response is formed by cells:


A. T-lymphocytes and B-lymphocytes

Downloaded by AMAN SINGH (amansinghgkp8@gmail.com)


lOMoARcPSD|19252957

B. Erythrocytes and leukocytes


C. Phagocytes
D. Platelets
E. Reticular-endothelial system

338. Nonspecific cellular factors of protection include:


A. Complement
B. Phagocytosis
C. Antibodies
D. Interferon
E. Lysozyme

339. Lysozyme:
A. Contained in serum, saliva
B. Does not have a bactericidal effect.
C. Participates in the reaction of phagocytosis
D. Stimulates antibody production
E. Has a lipid nature

340. Nonspecific resistance factor - lysozyme damages:


A. Cytoplasmic membrane
B. Cytoplasm
C.Nucleoid
D. Cell wall
E. Capsule

341. For the formation of passive immunity in the body enter:


A. Immunoglobulins
B. Live vaccines
C. Killed vaccines
D. Toxoid
E. Antigens

342. Antitoxic immunity is created when introduced into the body:


A. Killed microbes
B. Living microbes
C. Toxoid
D. Viruses
E. The simplest

Downloaded by AMAN SINGH (amansinghgkp8@gmail.com)


lOMoARcPSD|19252957

343. Specific immunity factors are:


A. Neutrophils
B. Interferon
C. Lysozyme
D. Complement
E. Immunoglobulins

344. Actively acquired immunity is formed in the body after:


A. Introduction serum
B. After disease
C. Blood Transfusions
D. Immunoglobulin administration
E. Antibiotic Therapy

345. Humoral mechanisms of non-specific protection:


A. Immune diagnostic serum
B. B-lysine
C. Lysozyme
D. Interferon
E. It is true all of the above.

346. With the introduction of anti-diphtheria serum into the human body,
immunity is formed:
A. Artificial passive
B. Artificially active
C. Natural active
D. Natural Passive
E. Antibacterial

347. Natural active immunity is formed after:


A. After disease
B. Vaccination
C. Administration of antibacterial immune sera
D. Immunoglobulin
E. Interferon

Downloaded by AMAN SINGH (amansinghgkp8@gmail.com)


lOMoARcPSD|19252957

348. Immunobiological preparations for the creation of active artificial


immunity: A. Immune sera
B. Interferons
C. Toxoid
D. Antitoxins
E. Immunoglobulins

349. Associated vaccine contains microorganisms:


A. Genetic engineering
B. One type
D. Inactivated
C. Multiple species
E. Living

350. Specific desensitization is carried out to prevent allergic reactions:


A.Infectious
B. Medicinal
C. Food
D. Cryogenic
E. Anaphylactic

351. Antitoxic serum contains antibodies to:


A.Bacteria
B. Endotoxins
C. Exotoxin
D. Virus
E. Prions

352. Immuno-biological preparations for the creation of active artificial


immunity:
A. Immunoglobulin
B. Hyperimmune Serum
C. Vaccine
D. Adjuvants
E. Interferon

353. Antitoxic sera cause:


A. Agglutination of bacteria, precipitation of antigens
B. Inhibition of hemagglutination

Downloaded by AMAN SINGH (amansinghgkp8@gmail.com)


lOMoARcPSD|19252957

C. Neutralizing toxin
D. White blood cell lysis
E. Red cell lysis

354. Formation of acquired immunity due to:


A. Inflammatory process
B. The action of lysozyme and complement
C. Reactions of T-and B-lymphocytes
D. Cell reactivity
E. Development of complete phagocytosis

355. Secret Ig class A immunoglobulins:


A. Interfere with the adhesion of microorganisms to epithelial cells of mucous
membranes
B. Activate the immune response
C. Complete phagocytosis
D. Accumulate in serum
E. Stimulate virus reproduction

356. When haemolytic disease in newborn may occur?


A. An Rh negative mother carries an Rh positive fetus
B. A Rh positive mother carries an Rh negative fetus
C. A Rh positive mother carries an Rh positive fetus
D. None of the above
E. Both of the above

357. Chemical mediators released during type I hypersensitivity reactions may


be:
A. Histamine
B. Serotonin
C. Eosinophil chemotactic factors of anaphylaxis
D. All of the above
E. None of the above

Clinical bacteriology
358. Which disease is caused by Gram negative cocci?
A. Rheumatism
B. Diphtheria

Downloaded by AMAN SINGH (amansinghgkp8@gmail.com)


lOMoARcPSD|19252957

C. Tuberculosis
D. Dysentery
E. Gonorrhea
359. Which disease is caused by Gram+ diplococci?
A. Pneumonia
B. Plague
C. Tularemia
D.Borreliosis
E.Gonorrhea
360. Which disease is caused by streptococci?
A.Measles
B. Scarlet fever
C.Ornithosis
D. Herpes
E. Rubella
361. What are the main methods used to diagnose staphylococcal infection?
A. Serological
B. Bacterioscopy
C. Allergic
D. Bacteriological
E. Biological
362. For which pathogen is hemolysin, lecithinase, coagulase production a
differential sign?
A. Str. Pyogenes
B. Staph. Epidermidis
C. N. meningitides
D. Staph. Aureus
E. Staph. Saprophyticus

363. What method help to determine the source of staphylococcal


nosocomial infection?
A. Antigenecity determination
B. Phagotyping
C. Enzyme activity determination
D. Toxygenecity determination
E. Determination of pathogenic enzymes

364. How does staphylococcal nosocomial strains differ from other


strains?
A. By morphology
B. By cultivation
C. By multidrug resistance

Downloaded by AMAN SINGH (amansinghgkp8@gmail.com)


lOMoARcPSD|19252957

D. By biochemical activity
E. By antigenic properties
365. Which is morphological characteristic of pathogenic staphylococci?
A. Angled position
B. Arranged in pairs
C. Arranged in long chains
D. Arranged in short chains
E. Arranged in grape like clusters
366. What drugs are used for serotherapy of staphylococcal infection?
A. Broad spectrum antibiotics
B. Specific bacteriophage
C. Specific immunoglobulines
D. Specific antibodies
E. Toxoid
367. What are the main factors due to the pathogenecity of staphylococci?
A. Ability to produce spores
B. Ability to produce volutin grains
C. Ability to produce toxin
D. Ability to produce flagella
E. Ability to produce saccharolytic enzymes

368. What drugs are used for specific prevention of staphylococcal


infection?
A. Bacteriophages
B. Toxoid
C.Autovaccine
D. Antibiotics
E.Immunoglobulin

369. Which of the factors contributes to the development of staphylococcal


infection?
A. Morphological features
B. The state of macroorganismimmune system
C. The presence of flagella
D. The occurrence of autoimmune reactions
E. Age features

370. What are volutin grains?


A. Water supply
B. Stock of toxins
C. Metabolite reserve
D. Nutrient supply
E. Enzyme reserve

Downloaded by AMAN SINGH (amansinghgkp8@gmail.com)


lOMoARcPSD|19252957

371. What morphological features are characteristic of the


CorynebacteriumDiphteria?
A. Contains capsule
B. Contains a dispute
C. Contains chitin
D. Contains volutin grains
E. Contains flagella
372. How does Corynebacterium diphtheria arranges in direct microscopy?
A. As a chain
B. At an angle
C. In the form of roman numerals
D. In grape like clusters
E.As a single cells
373. What is the main specimen for the diagnosis of diphtheria?
A. Feces
B. Urine
C. Pharyngeal mucus
D. CSF
E. Intestinal mucus

374. What is the main method for the diagnosis of diphtheria?


A. Allergic
B. Bacteriological
C.Virological
D. Genetic- engeneering
E. Biological

375. Who is the source of diphtheria?


A. Rodents
B. Insects
C. Human
D. Birds
E. Cold-blooded

376. What is transmission of diphtheria?


A. Aerosol
B. Sexually transmitted
C. Fecal- oral
d.Transmissive
e. Intrauterine

377. What types of the colonies is produced by causative agent of


diphtheria biovar gravis on tellurite- blood agar medium?
A. Reminiscent of droplets of mercury
B. Black colonies

Downloaded by AMAN SINGH (amansinghgkp8@gmail.com)


lOMoARcPSD|19252957

C. Pink colonies
D. Resembling fried eggs
E. Colorless colonies

378. How long does it takes to get diphtheria revaccination (by vaccine
DPT)?
A. 5 years
B. 7 years
C. 10 years
D. 6 years
E. 15 years

379. What is the basis for classification of the causative agent of


diphtheria into biovar gravis and mitis ?
A. Type of colonies
B. Biochemical reactions
C. Morphological features
D. Antigenic properties
E. Staining

380. What drug is used for specific passive prevention of diphtheria?


A. Toxoid
B. Vaccine
C. Antitoxin
D. Normal serum
E. Bacteriophage

381. Which vaccine is used for prevention of diphtheria?


A. BCG
B. 19-BA
C. EV
D. live vaccine
E. DPT

382. What drug is used for specific treatment of diphtheria?


A. Vaccine
b.Agglutinable vaccine
c. Antitoxin
d. Bacteriophage
e. Toxoid

383. Which infection disease is treated by antitoxin?


A. Brucellosis
B. Tularemia
C. Plague

Downloaded by AMAN SINGH (amansinghgkp8@gmail.com)


lOMoARcPSD|19252957

D. Tuberculosis
E. Diphtheria

384. What type of immunity is formed against diphtheria ?


A. Not persistant
B. Antitoxic
C. Antibacterial
D. Cell mediated
E. Passive

385. What is the morphological features of causative agent of diphtheria?


A. Volutin grains
B. Flagella
C. Capsule
D. Fimbriae
E. Spore
386. By what morphological basis is it possible to differentiate
Corynebacterium diphtheria from diphteroids?
A. By spore
B. By capsule
C. By flagella
D. By volutin grains
E. By fimbria
387. Which culture medium is selective for cultivation of bordetellae?
A. Blood agar
B. Casein agar
C. Chocolate agar
D. Bismuth sulphate agar
E. Sugar broth
388. What pathogenecity factors do B. pertussis have?
A.Thermolabile toxin
B. Adenylatecyclase
C.Fibrinolysin
D.Hyaluronidase
E.Hemaglutinin
389. What are the symptoms of whooping cough?
A.Clonic and tonic cramps
B. Conclusive cough
C. Meningeal symptoms
D. Bouts of fever
E. Profuse diarrhea
390. What specimen is used to diagnosis whooping cough?
A. Blood
B. Urine

Downloaded by AMAN SINGH (amansinghgkp8@gmail.com)


lOMoARcPSD|19252957

C. CSF
D. Mucus
E. Feces
391. What express diagnostic method is used for whooping cough?
A. Immunoflorescent
B. Allergic
C. Biological
D. Serological
E. Bacteriological
392. What drug is used for specific treatment of whooping cough?
A. Killed vaccine
B. Toxoid
C. Immunoglobulin
D. Bacteriophage
E. Antibiotic
393. What preparation is used for active specific prevention of whooping
cough?
A. Toxoid
B. Immunoglobulin
C. Live vaccine
D. Killed vaccine
E. Antitoxin
394. What type of mycobacteria belongs to the normal microflora of the
genital tract?
A. M. tuberculosis
B. M. bovis
C. M. avium
D. M. leprae
E. M. smegmatis
395. What morphological features characterize the causative agent of
tuberculosis?
A. Large size
B. Motility
C. Spore formation
D. Absence of capsule
E. Comma shape
396. What culture medium is used for cultivation of tubercle bacilli?
A. Mac Conkey
B. Hiss medium
C. Lowenstein-Jensen medium
d.Saburo
e. EMB
397. What type of colonies tubercle bacilli produced on Lowenstein-
Jensen medium?
A. Wrinkled and dry

Downloaded by AMAN SINGH (amansinghgkp8@gmail.com)


lOMoARcPSD|19252957

B. Smooth and moist


C. Looks like droplets of mercury
D. With smooth edges
E. Convex and colorless
398. By what properties M. tuberculosis differentiate from M. bovis?
A. Antibiotic sensitivity
B. Requirements for growth
C. Acid resistance
D. Character and growth time
E. Immunogenic properties
399. What is transmission of tuberculosis?
A. Aerosol
b. Transmissive
c. Parenteral
d. Fecal-oral
e. Contact
400. What is determined by microscopic examination of tuberculosis?
A. Serotype
b.Biovar
c. Acid resistance
d. Delayed hypersensitivity
e. Virulence factors
401. What express bacteriological method is used to diagnose
tuberculosis?
A. Cultivation on egg’s medium
b.Cultivation on citrate blood
c. Rabbit inoculation
d. Guinea pig inoculation
e. Cultivation on synthetic medium
402. Which vaccine is used for specific prophylaxis of meningococcal
disease?
A. Live
B. Chemical
C. Killed
D. Genetic engenering
E. Toxoid

403. What disease is caused by gonococci?


A. Rheumatism
B. Meningitidis
C. Trachoma
D. Blennorhea
E. Astma

Downloaded by AMAN SINGH (amansinghgkp8@gmail.com)


lOMoARcPSD|19252957

404. Which microorganisms causes blennorrhea?


A. Neisseria gonorrhoeae
B. Neisseria meningitides
C. Chlamydia trachomatis
D. Corynebacterium diphtheria
E. Neisseria mucosa

405. Which microorganisms does the fetus infect when passing through
the birth canal?
A. Staphylococcus
B. Pneumococcus
C. Gonococcus
D. Meningococcus
E. Enterococcus

406. Which microorganisms have the most pronounced antagonistic


properties in the human body?
A. Corynebacteria, mycobacteria
B. Mycoplasmas, chlamydia
C. Actinomycetes, fungi
D. Yersinia, francisella
E. Spirochetes, rickettsia

407. Which morphological property is characteristic of streptococci?


A. The rod shape form of cells
B. The arrangement in the form of short chains
C. The presence of spores
D. The presence of flagella
E. The presence of volutin granules

408. What are the properties of streptococci?


A. Unculturing
B. Antigenic heterogenecity
C. The formation of endotoxin
D. The ability to ferment lactose
E. Resistance to penicillin

409. What method is used to identify streptococci by antigenic structure?

Downloaded by AMAN SINGH (amansinghgkp8@gmail.com)


lOMoARcPSD|19252957

A. Infection of chicken embrio


B. Inoculation of mucus from the throat to the Mac Conkey medium
c. Inoculation of pathological material on the nutrient agar
d. Precipitation reaction
e. Agglutination reaction

410. What causes the appearance of skin rash with scarlet fever?
A. Enterotoxin
B. Exfoliatin
c.Eritrogenin
d. Hemolysin
e. Endotoxin

411. Which specimen is used for the microbiological diognosis of


pneumococcal infections?
A. Sputum
B. Urine
C. Blood
D. Feces
E. Mucus

412. What is the main method for the microbiological diagnosis of


pneumococcal infections?
A. Genetic engineering
B. Bacteriological
C. Skin and allergy test
D. DNA hybridization
E. Infection of chicken embryos

413. What are the pathogenicity factors of meningococcus?


A. Exotoxin
B. Capsule
C. Flagella
D. Fibrinolysin
E. Hemolysin

414. What is characteristic of meningococci?


A. The presence of flagella
B. The formation of spores

Downloaded by AMAN SINGH (amansinghgkp8@gmail.com)


lOMoARcPSD|19252957

C. Gram positive colour


D. Intracellular location
E. Anaerobic type of respiration

415. What are the properties of meninococcus?


A. Antigenic homogenecity
B. Sensitivity to t
C. Pathogenicity for animals
D. Growth on ordinary media
E. Formation of exotoxin

416. What is the transmission route for meningococcal infection ?


A. Inoculable
B. Alimentary
C. Airborne
D. Parenteral
E. Genital

417. What disease is caused by meningococcal infection?


A. Blenorrhea
B. Rheumatism
C. Inflamation of the meninges
D. Erysipelas inflammation
E. Delayed hypersensitivity

418. What is important for identification of meningococcus?


A. Carbohydrate fermentation
B. Ability to growth in saline media
C. Pigmentation
D. Ability to incomplete phagositosis
E. Ability to produse gas

419. What is the specimen for microbiological diagnosis of meningococcal


infection?
A. Sputum
B. Urine
C. Fesec
D. Saliva
E. Liquor

Downloaded by AMAN SINGH (amansinghgkp8@gmail.com)


lOMoARcPSD|19252957

420. What is the basis of microbiological diagnosis of meningococcal


infection?
A. Isolation of the pathogen in anaerobic conditions
B. Isolation of the pathogen in laboratory animals
C. Isolation and identification of pathogen
D. Allergy test
E. Skin test

421. What is the difference between pathogenic and saprophytic Neisseria?


A. Growth on nutrient agar
B. Fermentation of sucrose
C. Growth on serum agar at 22 t
D. Growth on nutrient broth
E. Fermentation of lactose

422. What is the diagnostic value for meningococcal infection?


A. Study of biochemical activity
B. Skin allergy test
C. Biological test
D. Antibody titer increase
E. Cell culture infection

423. Which vaccine is used for specific prophylaxis of meningococcal


disease?
A. Live
B. Chemical
C. Killed
D. Genetic engenering
E. Toxoid

424. What disease is caused by gonococci?


A. Rheumatism
B. Meningitidis
C. Trachoma
D. Blennorhea
E. Astma

425. Which microorganisms causes blennorrhea?

Downloaded by AMAN SINGH (amansinghgkp8@gmail.com)


lOMoARcPSD|19252957

A. Neisseria gonorrhoeae
B. Neisseria meningitides
C. Chlamydia trachomatis
D. Corynebacterium diphtheria
E. Neisseria mucosa

426. Which microorganisms does the fetus infect when passing through the
birth canal?
A. Staphylococcus
B. Pneumococcus
C. Gonococcus
D. Meningococcus
E. Enterococcus
427. Who discovered the causative agent of gonorrhea?
A. L. Pasteur
B. R.Koch
C. A.Neisser
D. D.Ivanovski
E. I.Mechnikov

428. What is characteristic of the causative agent of gonococci?


A. Gram positive coloration
B. Twisted form
C. Rod shaped form
D. Extra cellular location
E. Intra cellular location

429. What is characteristic of gonococci?


A. Resistance in nature
B. High enzymatic activity
C. Demanding for nutrient media
D. Sensitivity to optokhin
E. Pathogenecity to animals

430. How is gonorrhea transmitted?


A. Inoculable
B. Alimentary
C. Aerogenic
D. Sexual

Downloaded by AMAN SINGH (amansinghgkp8@gmail.com)


lOMoARcPSD|19252957

E. Transplacentary

431. Which materials are examined for the microbiological diagnosis of


gonorrhea?
A. Examine of feces
B. Examine of ascites fluid
C. Examine of spiral cord
D. Examine of discharge of the urethra, vagina
E. Examine of mucus from stomach

432. To identification of what bacteriological method used ?


A. To identify spores, capsules, flagella
B. To determine antibodies in blood, serum of patient
C. To determine the morphology of the bacterial cell
D. To determine the number of microbes in the test material
D. To detect antigens in the blood serum

433. Which acute infection can be diagnosed with direct microscopy


method?
A. In case of dysentery
B. To diagnosis of rickettsiosis
C. In case of mycoplasmosis
D. In case of gonorrhea
E. For diagnosis of whooping cough

434. How does the biological properties of Chlamydia can be studied?


A. Gram positive strains
B. Spore forming
C. Obligate intracellular parasite
D. Cultivated on simple nutrient agar
E. Not sensitive to antibiotics

435. What is the basis for the microbiological diagnosis of gonorrhea?


A. Microscopy of pathological material
B. Infection of laboratory animals
C. Inoculation of pus on the Hiss medium
D. Allergic test
E. Hemolysis reaction

Downloaded by AMAN SINGH (amansinghgkp8@gmail.com)


lOMoARcPSD|19252957

436. What is the diagnostic sign for gonorrhea?


A. Ability to complete phagocytosis
B. Incomplete phagocytosis
C. Convoluted form of pathogen
D. Pathogenicity for guinea pigs
E. Detection of gram positive cocc

437. Which microorganisms are causative agent of urethritis, which are not
clinically indistinguishable from gonorrhea?
A. Treponemapallidum
B. Chlamydia trachomatis
C. Brucellaabortus
D. Mycoplasma hominis
E. Francisellatularensis

438. How is the biological properties of chlamydia studied?


A. Infection of the chicken embryo
B. Crushed drop
C. Dark field microscopy
D. Break down of carbohydrates on Hiss medium
E. Culturing on the blood agar

439. What are the transmission routes of chlamydial urethritis?


A. By arthropods
B. it is zoonotic diseases
C. By contact way
D. Is rare in frequency
E. Present in normal flora

440. What is the basis for diagnosis of urogenital chlamydia?


A. By clinical picture of diseases
B. The presence of elementary reticular bodies in the cells of the
cylindrical epithelium of the urethra
C. Negative skin allergic test
D. Increase in hypersensitivity of the immediate type
E. The characteristics of growth on blood agar

441. Which method is diagnostic important for detection of mycoplasma?


A. Bacterioscopical examination of pus, mucus from the urethra

Downloaded by AMAN SINGH (amansinghgkp8@gmail.com)


lOMoARcPSD|19252957

B. Character of colonies on a solid nutrient medium


C. study of cerebrospinal fluid
D. Biochemical activity
E. Biological test

442. What some staphylococcal strains are capable to form?


A. Form spores
B. Excrete my colic acid
C. Form L-formation
D. Form flagella
E. Form lipopolysaccharide

443. Which culture medium is used for isolation of staphylococcus from


pus, exudate, sputum?
A. Casein- carbon agar
B. Yolk- salt agar
C. Nutrient broth
D. Serum telluride agar
E. Coagulated serum

444. Which nutrient medium is used for detection of pathogenically


significant sign of st.aureus?
A. Mac Conkey`s agar
B. Serum agar
C. Yolk salt gar
D. Peptone meat broth
E. Hiss medium

445. What causes the appearance of skin rash with scarlet fever?
A. Enterotoxin
B. Exfoliatin
c. Eritrogenin
d. Hemolysin
e. Endotoxin

446. Which specimen is used for the microbiological diognosis of


pneumococcal infections?
A. Sputum
B. Urine

Downloaded by AMAN SINGH (amansinghgkp8@gmail.com)


lOMoARcPSD|19252957

C. Blood
D. Feces
E. Mucus

447. What is the main method for the microbiological diagnosis of


pneumococcal infections?
A. Genetic engineering
B. Bacteriological
C. Skin and allergy test
D. DNA hybridization
E. Infection of chicken embryos

448. What are the pathogenicity factors of meningococcus?


A. Exotoxin
B. Capsule
C. Flagella
D. Fibrinolysin
E. Hemolysin

449. What is characteristic of meningococci?


A. The presence of flagella
B. The formation of spores
C. Gram positive colour
D. Intracellular location
E. Anaerobic type of respiration

450. What are the properties of meninococcus?


A. Antigenic homogenecity
B. Sensitivity to t
C. Pathogenicity for animals
D. Growth on ordinary media
E. Formation of exotoxin

451. What is the transmission route for meningococcal infection ?


A. Inoculable
B. Alimentary
C. Airborne
D. Parenteral
E. Genital

Downloaded by AMAN SINGH (amansinghgkp8@gmail.com)


lOMoARcPSD|19252957

452. What disease is caused by meningococcal infection?


A. Blenorrhea
B. Rheumatism
C. Inflamation of the meninges
D. Erysipelas inflammation
E. Delayed hypersensitivity

453. What is important for identification of meningococcus?


A. Carbohydrate fermentation
B. Ability to growth in saline media
C. Pigmentation
D. Ability to incomplete phagositosis
E. Ability to produse gas

454. What is the specimen for microbiological diagnosis of


meningococcal infection?
A. Sputum
B. Urine
C. Fesec
D. Saliva
E. Liquor

455. What is the basis of microbiological diagnosis of meningococcal


infection?
A. Isolation of the pathogen in anaerobic conditions
B. Isolation of the pathogen in laboratory animals
C. Isolation and identification of pathogen
D. Allergy test
E. Skin test

456. What is the difference between pathogenic and saprophytic Neisseria?


A. Growth on nutrient agar
B. Fermentation of sucrose
C. Growth on serum agar at 22 t
D. Growth on nutrient broth
E. Fermentation of lactose

457. What is the diagnostic value for meningococcal infection?

Downloaded by AMAN SINGH (amansinghgkp8@gmail.com)


lOMoARcPSD|19252957

A. Study of biochemical activity


B. Skin allergy test
C. Biological test
D. Antibody titer increase
E. Cell culture infection
Particular Bacteriology

458. A child developing fever, hypotension, erythema, & neck stiffness;


Which is the most likely toxin involved in this case?
A. Endotoxin
B. Erythrotoxin
C. Exotoxin
D. Neurotoxin
E. Botulinum toxin
459. A newborn baby brought to hospital with irritability, fever, & neck
rigidity; Which is the most common cause infection?
A. Group В streptococcus
B. Streptococcus pneumonia
C. Staphylococcus aureus
D. H. influenzae
E. N. meningitides
460. Which is the structure that is found in gram-negative bacteria but not
in gram-positive bacteria?
A. Capsule
B. Cell wall
C. Cytoplasmic membrane
D. Endospore
E. Outer membrane
461. Which of the following toxins continually stimulates adenylate
cyclase to overproduce cAMP by catalyzing the binding of ADP-
ribose to the Gs protein, leading to severe fluid loss?
A. Bordelella perfussive toxin
B. Cholera toxin
C. Clostridium bolulinum toxin
D. Diphtheria toxin
E. Tetanus toxin
462. What is drug of choice for pulmonary anthrax?
A. Penicillin
B. Ciprofloxacin
C. Erythromycin
D. Tetracycline
E. Ceftriaxone
463. A 17-years-old girl present on 6Ih day after onset of fever. She is likely

Downloaded by AMAN SINGH (amansinghgkp8@gmail.com)


lOMoARcPSD|19252957

to have typhoid fever. What is the best test for diagnosis would be?
A. Blood culture
B. Typhi dot test
C. Widal’s test
D. Bone marrow biopsy
E. Stool culture
464. A 9-years-old male presented with high-grade fever & lethargy since
last evening. Examination revealed petechial rashes on the skin.
Clinical diagnosis of meningitis was suspected & lumbar puncture was
performed. CSF examination showed WBC l x 109/L with 94%
neutrophils. No organisms were seen on gram staining. Blood cultures
from outside hospital were subsequently positive for oxidase positive,
gram-negative diplococci. Which bacterium is most likely to be
causing this patient’s illness?
A. E. coli
B. Hemophilus influenza
C. Neisseria meningitidis
D. Pseudomonas aeroginosa
E. Streptococcus pneumoniae

465. A 10-year-old boy had meningitis; What is the causative organism?


A. Klebsiella
B. E. coli
C. Neisseria meningitides
D. Staphylococcus aureus
E. Salmonella
466. A previously healthy 3-month-old boy is brought to the physician by
his parents because he continues to have fever & ear pain despite
treatment with amoxicillin for 72 hours. Examination shows an
immobile, red, & opaque tympanic membrane. Which of the following
is the most likely pathogen?
A. Chlamydia trachomatis
B. Group A streptococcus
C. Hemophilus influenzae
D. Mycoplasma pneumoniae
E. Staphylococcus aureus
467. Which of the following disease doesn't cause ulcers of penis?
A. Gonorrhea
B. Syphilis
C. Lymphogranuloma venereum
D. Granuloma inguinale
E. Herpes simplex genitalis
468. Which bacteria is Gram negative cocci?
A. Neisseria
B. Staphylococcal

Downloaded by AMAN SINGH (amansinghgkp8@gmail.com)


lOMoARcPSD|19252957

C. Streptococci
D.Pneumococcus
E. E.coli

469. Due to the what gonococci is virulent?


A. Pili
B. Enterotoxin
C. Exotoxin
D. None
E. All

470. Where does gonococci initially infect?


A. Vagina
B. Cervix
C. Uterus
D. Fallopian tubes
E. Throat

471. A 48-year-old man with alcoholism comes to the physician because of


fever, a facial rash, & rapidly progressive swelling of the left side of
the face. He is unable to open his left eye because of the severity of
the swelling. Which of the following is the most likely causal
organism?
A. Group A streptococcus
B. Hemophilus influenzae
C. Herpes simple virus
D. Neisseria meningitides
E. Streptococcus pneumoniae
472. A young previously healthy IV drug addict was admitted in the
hospital with the complaints of fever & malaise for the last five days.
From two sets of blood culture there was growth of gram-positive
cocci arranged in clusters. Which of the following organisms is most
likely to be responsible for the endocarditis in the case?
A. Staphylococcus
B. Streptococcus
C. Rhotococcus
D. Enterococcus
E. Streptococcus pneumoniae
473. A 45-years-old gentleman develops deformity of nose. The biopsy of
lesion shows granulomas. Which of the most likely diagnosis is?
A. M. tuberculosis
B. M. leprae
C. Nocardia
D. M. avium-intercellulare
E. Aspergilloma

Downloaded by AMAN SINGH (amansinghgkp8@gmail.com)


lOMoARcPSD|19252957

474. What type of immunity is formed in TB?


A. Sterile
B. Non sterile
C. Humoral
D. Antitoxic
E. Natural
475. What preparation is used for Mantoux test?
A. Antraxin
B. Tularin
C. Tuberculin
D. Toxoplasmin
E. Lepromin
476. What vaccine is used for prevention TB?
A. OPV
B. TT
C. ВА-19
D. BCG
E. DPT
477. Which animal is used for cultivation of M. leprae?
A. Rabbit
B. Mice
C. Monkey
D. Armadillo
E. Guinea pig

478. What is morphological characteristics of M. leprae?


A. Comma shaped
B. Motile
C. Drum stick appearance
D. Straight or slightly curved bacilli
E. Straight with rounded ends bacilli
479. What is the arraignment of leprae bacilli in affected tissues?
A. In chains
B. Grape like clusters
C. In pairs
D. Masses of “globi” or “cigar bundle”
E. Like “Chinese letter”
480. Which is tinctorial property characterize leprae bacilli?
A.Resistance to acids
B. Resistance to phages
C. Resistance to heat
D. Resistance to antibiotics
E. Resistance to antiseptics
481. What method of diagnosis is used for leprosy?
A. Clinical method

Downloaded by AMAN SINGH (amansinghgkp8@gmail.com)


lOMoARcPSD|19252957

B. Biochemical method
C. Microscopical method
D. Epidemiological method
E. Cultivation method
482. What sample is used for diagnosis of leprosy?
A. Sputum
B. Feces
C. Affected tissues
D. Urine
E. CSF
483. What kind of immune response is the basis for allergic method of diagnosis
in leprosy?
A. Antibody – mediated
B. Cell – mediated
C. Anti – toxic
D. Innate
E. Herd
484. How do we distinguish between M.tuberculosis and M.leprae?
A. By acid fastness
B. By growth on culture media
C. By types of respiration
D. By antibiotic resistance
E. By biochemical activity
485. What kind of sample is necessary to obtain in actinomycosis?
A. Мокрота
B. Гной
C. Моча
D. Ликвор
E. Слизь из зева
486. What type of culture media is used for cultivation of actinomycetes?
A. Sabouraud’s media
B. MacConkey’s media
C. Lowenstein Jensen media
D. Loeffler’s media
E. McLeod’s media
487. Which does type of bacteria belong to the family of Enterobacteriaceae?
A. Staphylococci
B. Bacillus
C. Corynebacteria
D. Bordetella
E. Proteus
488. What genus do coliforms belong?
A. Shigella
B. Klebsiella
C. Salmonella

Downloaded by AMAN SINGH (amansinghgkp8@gmail.com)


lOMoARcPSD|19252957

D. Escherichia
E. Yersinia
489. What are the morphological signs characterize E. coli?
A. Presence of flagella
B. Absence of flagella
C. Spore formation
D. Chopped ended
E. Arranged in chains
490. Which microorganisms are capable to form capsule?
A. Salmonella
B. Escherichia
C. Shigella
D. Vibrio
E. Klebsiella
491. What is the importance of normal flora?
A. Does not cause inflammation in the main biotope
B. Does not cause severe form of infection
C. Antagonist to pathogenic microbes
D. Does not develop antibiotic resistance
E. Does not produce exotoxin
492. What types of culture media is used for isolation of E.coli?
A. Robertson’s cooked meet media
B. Lowenstein Jensen’s media
C. Loeffler’s media
D. MacConkey’s media
E. Wilson and Blair media
493. What is significant property of coliforms?
A. Gram positive
B. Require alkaline media
C. Lactose fermentation
D. Spore formation
E. Presence of volutin granules
494. What kind properties are differentiate opportunistic-pathogenic and
enteropathogenic E.coli?
A. Morphological features
B. Biochemical activity
C. Antigenic structure
D. Cultural features
E. Staining features
495. What is the basis for isolation of pure culture of enteropathogenic E.coli
from MacConkey’s media?
A. Slide agglutination reaction
B. Precipitation reaction
C. Hemolysis reaction
D. Neutralization reaction

Downloaded by AMAN SINGH (amansinghgkp8@gmail.com)


lOMoARcPSD|19252957

E. Indirect hemagglutination reaction


496. What properties are used to differentiate coliforms and salmonella?
A. Morphological
B. Staining
C. Biochemical
D. Toxigenecity
E. Antigenicity
497. What is controlling toxin production of E.coli?
A. Prophage
B. Pili
C. Capsule
D. Plasmid
E. Transposon
498. What do S. typhi, S. paratyphi A and B morphological properties have?
A. Peritrichous
B. Monotrichous
C. Lophotrichous
D. Amphitrichous
E. Do not have flagella
499. What property does combine typhoid salmonella and paratyphoid A and B?
A. Fastidious to culture media
B. Ferment lactose
C. Gram positive
D. Obligate anaerobes
E. Motility
500. What is the basis of Kauffmann-White’s classification of salmonella?
A. Biochemical activity
B. Antigenic structure
C. Staining properties
D. Morphological features
E. Toxin production
501. What method is used for microbiological diagnosis of salmonella?
A. Microscopy
B. Cultivation
C. Animal inoculation
D. Molecular
E. Skin test
502. What is differential property between Salm. typhimirium и Salm.
Enteritidis?
A. Morphology
B. Cultural characteristics
C. Carbohydrate metabolism
D. Protein metabolism
E. Antigenic properties
503. What properties are differentiate Shigella types?

Downloaded by AMAN SINGH (amansinghgkp8@gmail.com)


lOMoARcPSD|19252957

A. Morphological features
B. Gram staining
C. Antigenic structure
D. Staining property
E. Production of H2S
504. Which infection is characterized with fecal-oral mechanism of
transmission, affection of large intestine, tenesmus, presence of
mucous and blood on stool?
A. In Typhoid fever
B. In Cholera
C. In Leptospirosis
D. In Dysentery
E. In Relapsing fever
505. Who is a reservoir of dysentery?
A. Infected patients
B. Poultry
C. Rodents
D. Cattle
E. Small cattle
506. Какие морфологические свойства характерны для холерных
вибрионов ?
A. Monotrichous
B. Straight cells
C. Lophotrichous
D. Capsule formation
E. Peritirchous
507. What are the cultural characteristics of V.cholrae?
A. Grow on alkaline media and form smooth transparent S-colonies
B. Anaerobic type of respiration
C. Salt-loving
D. Fastidious to culture media
E. Slow growing
508. What kind of properties are used to differ cholera and cholera-like vibrions?
A. Morphology
B. Gram staining
C. Antigenic structure
D. Culture properties
E. Capsule formation
509. Which factor does diarrhea cause in cholera?
A. Presence of flagella
B. Synthesis of mucinases
C. Slightly alkaline environment
D. Action of exotoxin
E. Action of endotoxin
510. What preparation is used for specific prevention of cholera?

Downloaded by AMAN SINGH (amansinghgkp8@gmail.com)


lOMoARcPSD|19252957

A. Antitoxic serum
B. Antibacterial serum
C. Toxoid
D. Immunoglobulin
E. Interferon
511. What is express diagnosis of cholera?
A. Immfluorescent
B. Skin test
C. Animal inoculation
D. Agglutination reaction
E. Culture on alkaline media
512. Which vaccine is used for prevention of cholera?
A. Live
B. Killed
C. Genetically engineered
D. Chemical
E. Subunit
513. What are specific clinical characteristics of cholera?
A. Bacterial sepsis
B. Skin rashes
C. Spasmodic coughing
D. Dehydration
E. Clonic and tonic convulsions
514. Which infection’s pathogen does belong to genus Clostridium?
A. Diphteria
B. Tetanus
C. Tuberculosis
D. Cholera
E. Dysentery
515. What is morphological characteristic of Cl.tetani?
A. Terminal spore
B. Volutin granules
C. Presence of capsule
D. Absence of spores
E. Gram negative staining

516. A 43-years-old patient in hospital acquires pseudomonas


infection;Which of the most likely pathogenesis involved in it is?
A. Endotoxin
B. Activation of cAMP
C. Activation of EF-2
D. Exotoxin
E. Inhibition of cGMP
517. A patient comes to you with cervical lymphadenopathy, &
histopathology of the node shows granulomatous inflammation. For

Downloaded by AMAN SINGH (amansinghgkp8@gmail.com)


lOMoARcPSD|19252957

ТВ diagnosis which is most diagnostic?


A. Mantoux test
B. PCR
C. Caseation necrosis
D. AFB
E. X-ray chest
518. The TSST-1 toxin of staphylococcus aureus acts to produce
disseminated intravascular coagulation & circulatory collapse by
functioning as a(n)
A.Inducer of MHC antigen expression
B.Polyclonal В cell activator
C.Producer of endogenous pyrogens
D.Superantigen
E.T cell lectin
519. Regarding actinomycosis which of the following arenot true?
A. Characterized by presence of sulphur granules in pus.
B. Characterized by abscess formation.
C. Caused by an anaerobic organism.
D. Characterized by draining sinus tracts.
E. Transmitted by personal contact.
520. Due to the what gravity of diphtheria is?
A. Toxin production
B. Dehydration
C. Hyperpyrexia
D. Bleeding
E. Pseudomembrane formation
521. A 71-year-old male patient has recent aggravation of an exfoliative skin
condition. He now has a temperature of 102 degrees F. The skin of his upper
chest, extremities, & neck shows erythema with diffuse epidermal peeling &
many pustular lesions. Cultures of pus from the lesions yields a gram-positive
organism that is highly salt (NaCl) tolerant. Which of the following laboratory
data would be most helpful for the identification of this microorganism?
A. Bacitracin sensitivity
B. Bile solubility
C. Coagulase positivity
D. Optochin sensitivity
E. Oxidase positivity
522. By what is related virulence of bacteria?
A. Toxin & enzyme production
B. Resistance of the patient
C. Number of bacteria
D. Age of the patient
E. Portal of entry
523. How brucellosis is transmitted?
A. Flies

Downloaded by AMAN SINGH (amansinghgkp8@gmail.com)


lOMoARcPSD|19252957

B. Unboiled milk
C. Cats
D. Dogs
E. Feco-oral route
524. Which of the following is true regarding pseudomonas?
A. Sensitive to tetracycline
B. Resistant to quinolones
C. Cause malignant otitis externa
D. Always complicated by brain abscess
E. Most common cause of infection in childhood
525. How spores will be killed?
A. 121°C, at 15 lb/in2, for 15 minutes (moist heat)
B. 73“C for 2 hours (dry heat)
C. Pasteurization
D. Chlorination
E. iodine application
526. A sexually active woman develops pain in lumbar region with fever, &
on asking she also mention about discoloration of urine; What will be
the next step?
A. Blood culture
B. Blood & urine culture
C. Urine culture
D. Cystoscopy
E. Urine D/R
527. Which bacteria is caused Gas gangrene?
A. Clostridia botulinum
B. Clostridia tetani
C. Clostridia perfringens
D. Vibrio cholera
E. Proteus
528. Which of the following is the most likely immunization against
tetanus?
A. Anti-toxin
B. Tetanus toxoid
C. Toxin
D. Immunoglobulin
E. Penicillin
529. What is caused abscess containing sulphur granules?
A. Actinomycosis
B. Staphylococcus aureus
C. Brucellosis
D. Amebiasis
Е. Histoplasmosis
530. Which of the following is true regarding
corynebactenum diphtheria?

Downloaded by AMAN SINGH (amansinghgkp8@gmail.com)


lOMoARcPSD|19252957

A. Needs nutrient agar


B. Shows pleomorphism
C. Is a gram +ve bacilli
D. Show complete alpha-hemolysis
E. Produces mainly endotoxin
531. Which bacteria doesn't produce exotoxin?
A. S. dysenteriae.
B. S. typhi.
C. V. cholera.
D. C. diphtheria.
E. Clostridium tetani.
532. A child has presented with ear infection; pus is greenish in color. What
is the probable organism?
A. Pseudomonas aeruginosa
B. Staphylococcus aureus
C. Streptococcus pyogenes
D. Streptococcus pneumoniae
E. Klebsiella
533. A patient has fever & murmur due to valvular heart disease; which
bacteria is the causative agent?
A. Streptococcus viridans
B. Staphylococcus aureus
C. H influenza
D. E coli
E. Streptococcus pneumoniae

534. A child develops pharyngitis caused by group A beta-hemolytic


streptococci. Five weeks following treatment, he develops impetigo, &
another group A beta-hemolytic streptococcus is isolated. These
results are best explained by which of the following?
A. Immunity to streptococcal disease is type- specific
B. Immunity to streptococcus is organ- specific
C. The first culture was misidentified
D. The patient did not develop immunity to the initial streptococcus infection
E. All above of them
535. A neonate develops meningitis. Streptococcus is isolated from the
mother’s vagina. The organism agglutinates with antiserum directed
against type В surface carbohydrate. The virulence of this organism is
related to a bacterial constituent that interferes with which of the
following host phagocyte functions?
A. Aggregation
B. Chemotaxis
C. Ingestion
D. Intracellular killing
E. Pseudopod formation

Downloaded by AMAN SINGH (amansinghgkp8@gmail.com)


lOMoARcPSD|19252957

536. Which bacteria caused Neonatal meningitis?


A. E coli
B. H influenzae
C. Meningococci
D. Streptococcus pneumonia
E. Staphylococcus aureus
537. A 25-years-old lady had fever, & sore-throat. On culture of throat
swab, organism causes complete hemolysis of culture medium. What
is the most likely organisms?
A. a-hemolytic streptococcus
B. β-hemolytic streptococcus
C. Bacteroides
D. Viruses
E. E. coli
538. In a hospital catalase +ve staphylococcal infection outbreak occur;
Which of the following is least likely to be done?
A. Sterilization of all contacts
B. Isolation of organism
C. Isolation of further sub-type
D. Isolation of patient
E. Vancomycin prophylaxis of all contacts
539. A child has presented with throat infection to ENT ward. Swab from
the throat was cultured, which showed complete hemolysis; which will
be the most probable organism?
A. Beta hemolytic streptococci
B. Staphylococcus aureus
C. Mycoplasma
D. Klebsiella pneumoniae
E. Streptococcus viridans
540. What is most important for diagnosis of ТВ?
A. Caseous necrosis
B. Granuloma
C. Tubercle bacilli
D. Langhan’s giant cell
E. Epithelioid aggregation
541. Which of the following test is used for virulent treponema pallidum?
A. Treponema pallidum immobilization test.
B. Reiter protein complement fixation test.
C. Wassermann complement fixation test.
D. Fluorescent treponemal antibody test.
E. Fluorescent treponemal antibody/ absorption test.
542. A patient has symptoms & signs of tuberculosis; what will be the
confirmatory test?
A. AFB

Downloaded by AMAN SINGH (amansinghgkp8@gmail.com)


lOMoARcPSD|19252957

B. Granuloma
C. Macrophages
D. Urine
E. Stool
543. Which of the following laboratory tests is useful for the diagnosis of
Lyme disease?
A. Blood culture on sheep blood agar plate
B. Detection of IgM/lgG antibodies to the spirochete
C. Detection of specific antibody to ixodes tick
D. Documentation of fever & arthritis
E. Spinal fluid culture on Thayer-Martin agar
544. Where does actinomycetes can’t cause disease?
A. Bone
B. Lung
C. Brain
D. Skin
E. Kidney -

545. Which is the least titer for Vidal test to be positive?


A. 80
B. 120
C. 160
D. 320
E. 20
546. A young boy is having swelling & tenderness of the knee joint; What
is the most likely pathogen involved?
A. Streptococcus pyogenes
B. Staphylococcus aureus
C. Gonococcus
D. Brucella
E. Mycobacterium tuberculosis
547. When compared to a non-pathogenic strain of Corynebacterium
diphtheria, pathogenic C. diphtheriae possess?
A. a temperate bacteriophage
B. an epitope
C. an F-plasmid
D. bacteriocin
E. purified DNA from a eukaryotic cell
548. In the first week of typhoid fever which is the most probable
diagnostic test?
A. Widal test
B. Typhi dot
C. Urine culture
D. Stool culture
E. Blood culture

Downloaded by AMAN SINGH (amansinghgkp8@gmail.com)


lOMoARcPSD|19252957

549. Approximately 35 people attending a picnic developed symptoms of


food poisoning, which occurred approximately 8 hours after everyone
ate lunch. Laboratory studies revealed that the causative agent is a
microaerophilic gram- negative, curved rod with polar flagella, & a
“seagull “or “comma” appearance. Which of the following is the most
likely reservoir for this organism?
A. Fish
B. Fried rice
C. Improperly canned food
D. Poultry
E. Vegetables
550. Which of the following toxins stimulates adenylate cyclase by
catalyzing the transfer of ADP-ribose to the inhibitory subunit of the G
protein?
A Bordetella pertussis toxin
B Clostridium botulinum toxin
C Clostridium tetani toxin
D Corynebacterium diphtheria toxin
E Staphylococcus aureus toxin

551. A 37-year-old woman with a history of rheumatic fever recently had a


dental extraction. She did not take prophylactic antibiotics. She now
presents with a low-grade fever, malaise, & a cardiac murmur.
Echocardiography is positive for a valvular lesion. Blood cultures are
positive. Which of the following is the most likely causative
microorganism?
A. Coxsackie virus
B. Enterococcus faecalis
C. Pseudomonas aeruginosa
D. Staphylococcus aureus
E. Viridans streptococci

552. Which bacteria caused boil of the nose ?


A. Streptococcus pneumonia
B. Streptococcus pyogenes
C. Staphylococcus aureus
D. Pseudomonas
E. Proteus

553. An 18-year-old student complains of fever, cough, chest pain, &


weight loss. On physical exam, adenopathy is noted. Lymph node
biopsy reveals caseating granulomatous inflammation. Adjacent to the
areas of necrosis, macrophages are seen, filled with multiple small
oval structures that appear to have halos. This student most likely
acquired his infection from which of the following sources?

Downloaded by AMAN SINGH (amansinghgkp8@gmail.com)


lOMoARcPSD|19252957

A. Another human via respiratory secretions


B. Cat feces contaminated drinking water
C. Contaminated drinking water
D. Desert sand
E. Soil enriched with bird excrement
554. A 15-day-old infant presents with purulent conjunctivitis. Inclusion
bodies are seen in conjunctival scrapings. Which of the following is
the infectious form of the organism?
A. Arthrospore
B. Conidia
C. Elementary body
D. Endospore
E. Reticulate body
555. With an appropriately performed acid-fast staining procedure, To
which color mycobacteria will appear?
A. blue
B. brown
C. colorless
D. purple
E. red
556. A 43-year-old man presents to his physician complaining of a sore on
his penis. He admits that he visited a prostitute three weeks ago.
Physical examination reveals a single firm, raised, red, non-tender
lesion at the base of his penis His physician provisionally diagnoses
primary syphilis. Which of the following laboratory procedures would
be most likely to verify the diagnosis?
A. Culture on Fletcher’s serum semi-solid medium
B. Gram’s stain
C. Immunofluorescent stain of smear made from the active lesion
D. Rapid plasma reagin (RPR) assay
E. Special culture using charcoal-yeast extract agar
557. A 7-year-old child has had a high fever & a sore throat for 2 days.
Examination shows pharyngeal erythema, a swollen right tonsil with a
creamy exudates, & painful right submandibular lymphadenopathy.
Throat culture on blood agar yields numerous small b- hemolytic
colonies that are inhibited by bacitracin. Which of the following is the
most likely causal organism?
A. Advenovirus
B. Candida albicans
C. Corynebacterium diphtheriae
D. Coxsackie virus
E. Streptococcus pyogenes (group A)

558. For which of the following diseases would strict isolation be indicated
for a hospitalized patient?

Downloaded by AMAN SINGH (amansinghgkp8@gmail.com)


lOMoARcPSD|19252957

A. Botulism
B. Cervical-facial actinomycosis
C. Mycobacterium kansasii pulmonary infection
D. Pneumococcal pneumonia
E. Y. pestis pneumonia
559. Enteric gram-negative bacteria are most resistant to penicillin G than
gram-positive bacteria. Which of the following is most closely
associated with this difference?
A. Cytoplasmic membrane
B. Lipoprotein
C. Outer membrane
D. Peptidoglycan
E. Teichoic acid
560. A 12-year-old girl with sickle cell disease has pain in her right arm. An
x-ray film of her arm shows bony lesions consistent with
osteomyelitis. Which of the following is the most likely causal
organism?
A. Clostridium septicum
B. Enerococcus faecalis
C. Listeria monocytogenes
D. Proteus mirabilis
E. Salmonella enteritidis
561. A 18 years old girl after eating CANNED food end up in emergency
department, with vomiting <,nd a blood pressure of 80 systolic, pale
end calm extremities with a pulse of 30 per minute. She is in a state of
confusion. Due tothe whatthe reaction probably?
A. Enterotoxin
B. Endotoxin
C. Preformed toxin
D. Exotoxin
E. None of above
562. During 2nd week of typhoid fever, which of the following will be the
most accurately diagnostic?
A. Widal test
B. Urine culture
C. Stool culture
D. Blood culture only
E. Blood culture + Vidal test

563. A married woman has right sided tubo-ovarian abscess, & she gave
history of IUCD; What is the causative organism?
A. Chlamydia
B. Gardenella
C. Tuberculous
D. Bacteroides

Downloaded by AMAN SINGH (amansinghgkp8@gmail.com)


lOMoARcPSD|19252957

E. Herpes simplex
564. Which of the following lesions doesn` t cause Staphylococcus?
A. Abscess.
B. Carbuncle.
C. Boil.
D. Cellulitis.
E. Lymphangitis.

565. A child is complaining of fever & wrist joint pain; what investigation
will help you to reach a diagnosis?
A. ASO titer
B. ESR
C. RA factor
D. Blood CP
E. X-ray wrist joint
566. A young girl came with signs & symptoms of & diagnosed as
meningitis. Which organism will be involved?
A. Hemophilus influenzae
B. N. meningitides
C. Staphylococcus aureus
D. Streptococcus pyogenes
E. Diphtheria
567. Due to the whatGram negative bacteria in hospitalized patient?
A. Indwelling urinary catheter
B. Pneumonia
C. 1/V cannula *
D. Broad spectrum antibiotics
E. Nil per oral
568. Regarding corynebacterium diphtheria all of the following are not
true?
A. Is an anaerobe.
B. Produces powerful exotoxin.
C. Is a gram positive bacillus.
D. Causes pseudomembranous inflammation of pharynx.
E. Is clubbed shaped.

569. Which of the following arenot gram -ve pathogenic diplococcic?


A. Neisseria meningococci.
B. Neisseria gonococci.
C. Neisseria lactamica.
D. Neisseria flavescens.
E. Streptococcus pneumoniae.

570. Which disease can’t be caused by Gram +ve cocci?


A. Impetigo.

Downloaded by AMAN SINGH (amansinghgkp8@gmail.com)


lOMoARcPSD|19252957

B. Boil.
C. Wool-sorters disease.
D. Scarlet fever.
E. Ludwig's angina.

571. In a 7 year old boy with sickle cell anemia, which of the following can
lead to septicemia?
A. E. coli
B. H. influenza
C. Pneumococci
D. Salmonella
E. Pseudomonas
572. Which organisms doesn`t cause UTI?
A. E. coli.
B. Proteus.
C. Bacteroides.
D. S. fecalis.
E. Pseudomonas.
573. Which bacteria is not aerobic or facultative anaerobic gram negative
rod that producing endotoxin?
A. E. coli.
B. Clostridia.
C. Proteus species.
D. Pseudomonas aeroginosa.
E. Klebsiella pneumonia.

574. A 26-years-old woman came to you with history of fever & weakness.
On examination temperature of 38.9°C (102°F), BP 130/70 mm Hg,
marked tachycardia & change of murmur was noticed. Which is the
most common cause of infective endocarditis?
A. Streptococcus aureus
B. Streptococcus pneumoniae
C. Streptococcus viridans
D. Group В beta hemolytic streptococcus
E. Group D streptococcus

575. Which of the following organisms caused food poisoning desease?


A. Clostridium perfringens.
B. Clostridium botulinum.
C. Salmonella.
D. B. cereus.
E. P. vivax.

576. What is a very frequent cause of hospital acquired infections?


A. Klebsiella

Downloaded by AMAN SINGH (amansinghgkp8@gmail.com)


lOMoARcPSD|19252957

B. Pseudomonas
C. S. aureus
D. H. influenzae
E. S. Pneumoniae

577. Which of the following serology tests not for syphilis?


A. VDRL.
B. Vidal.
C. TPI.
D. TPHA.
E. Wasserman

578. Which one is not intracellular parasite?


A. Brucella abortus.
B. R. prowazekii.
C. P. aeruginosa.
D. L. monocytogenes.
E. M. tuberculosis

579. Which one is not character of staphylococcus aureus?


A. Golden colony pigmentation.
B. The production of coagulase.
C. Gram –ve bacteria
D. The production of leukocidin.
E. The production of soluble hemolysin.

580. Which of the following bacteria are not commonly found in infected
wounds?
A. Escherichia coli.
B. Neisseria meningitidis.
C. Streptococcus anginosus.
D. Streptococcus fecalis.
E. Bacteroides fragilis.

581. Blood is drawn from a 14-year- old boy with bacterial meningitis for a
complete blood count. The leukocyte count is elevated. Which of the
following is released by the predominant type of white blood cell
present?
A. Histamine
B. Leukotriene
C. Vasoactive amines
D. Peroxidase
E. Lysozyme

582. Regarding bacterial meningococcal meningitis which of the following

Downloaded by AMAN SINGH (amansinghgkp8@gmail.com)


lOMoARcPSD|19252957

is not true?
A. CSF proteins are raised.
B. CSF blood sugar is decreased.
C. Bacteria can be cultured on media.
D. Bacteria can be gram stained.
E. CSF has no neutrophil.

583. A patient had puerperal pyrexia, what is the most likely causative
organism?
A. E. coli
B. Gonococci
C. Bacteroides
D. Mixed bacterial infection
E. Chlamydia

584. Which of the following route infection within a hospital may not occur
via?
A. Dust-borne.
B. Water-borne.
C. Food-borne.
D. Hand-borne.
E. Endogenous.
585. By which following organism folliculitis is caused?
A. Proteus.
B. Pseudomonas.
C. Klebsiella.
D. Staphylococci.
E. E. coli.
586. Regarding lymphogranuloma venereum all of the following are not
true?
A. Caused by chlamydia trachomatis.
B. Occurs more in women.
C. Most common sexually transmitted disease in USA.
D. Initial stages can be treated by tetracycline.
E. Surgery may be required in later stages.
587. Which of the following is true regarding lepromatous leprosy?
A. Dermis appears diffusely packed with vacuolated macrophages.
B. There is a high level of cell mediated immunity.
C. Mucous membranes are not involved.
D. The commonest cause of death is renal failure.
E. Lymphadenitis reaction.
588. Which of the following organ mostly affect Tuberculosis?
A. Ovary
B. Fallopian tubes
C. Uterus

Downloaded by AMAN SINGH (amansinghgkp8@gmail.com)


lOMoARcPSD|19252957

D. Breast
E. Vagina
589. A patient who is admitted in hospital with one week history of fever,
cough & sore-throat. Culture of throat swab on blood agar shows tiny
colonies with complete hemolysis of RBC's;What is the causative
organism?
A. Beta hemolytic streptococci
B. Hemophilus influenza
C. Staphylococcus
D. Bordetella pertussis
E. Corynebacterium diphtheria
590. Which bacteria can grow at 42°C?
A. Vibrio cholera
B. Pseudomonas
C. Shigella
D. E coli
E. Salmonella
591. What is the most common cause of puerperal infection?
A. Bacteroides
B. Candida
C. Trichomonas
D. Clostridia
E. HPV
592. A women comes to you with Siamese cat; which type of infection will
get her child?
A. Rubella
B. Syphilis
C. CMV
D. HPV
E. Toxoplasmosis
593. How spores should be killed?
A. Dry heat at 100°C
B. Dry heat at 60 °C
C. Dry heat at 160°C
D. Cidex solution
E. Moist heat
594. Which of the following is not a disinfectant?
A. Derivatives of salicylic acid
B. Alcohol
C. Soap
D. Hydrogen peroxide
E. Gluteraldehyde
595. A woman comes to you with pink purulent vaginal discharge; What
can be causative agent?
A. E. coli

Downloaded by AMAN SINGH (amansinghgkp8@gmail.com)


lOMoARcPSD|19252957

B. Chlamydia
C. Gonorrhea
D. Trichomonas
E. Giardia
596. What is the least positive titer for Vidal test?
A. 1:80
B. 1:160
C. 1:260
D. 1:320
E. 1:640
597. A child comes to you with recurrent fever & klebsiella infection; What
can be name of disease?
A. Chronic granulomatous disease of nose
B. Progressive atrophy of nasal mucosa
C. Nosocomial infection
D. Bacteremia
E. Hemorrhagic necrotizing consolidation of lung
598. Regarding the rickettsiae which of the following is not true?
A. Are morphologically pleomorphic.
B. Contain both RNA & DNA.
C. Multiply by binary fission.
D. Are gram-positive.
E. Are intracellular.
599. A patient came to you with sore-throat & fever; which test you will
advise to confirm the diagnosis?
A. Blood culture
B. Blood & sputum culture
C. ELISA of throat swab
D. Schultz Charlton reaction
E. В, C, D&E
600. Which of the following not produce Pseudomonas aeruginosa?
A. Exotoxin
B. Endotoxin
C. Enterotoxin
D. Hemolysin
E. Proteases

601. A 70-years-old gentleman dies of respiratory failure. Post mortem


biopsy of lung showed the evidence of histoplasmosis. Which structure
primarily involves this fungus?
A. Respiratory system
B. Central nervous system
C. Gastrointestinal system
D. Reticuloendothelial system
E. Genito-urinary system

Downloaded by AMAN SINGH (amansinghgkp8@gmail.com)


lOMoARcPSD|19252957

602. How mucor disease is diagnosed?


A. HPLC (high perfonnance liquid chromatography).
B. Myocolie assay.
C. Finding hypae.
D. Clinically
E. Ultrasound
603. Where does Candida albicans can’t colonize?
A. Gut.
B. Skin.
C. Vagina.
D. Mouth.
E. Prepuce.
604. A 37-year-old drug addict has white plaque¬like lesions on his tongue
& thru-out his mouth. The organism grows on Sabouratd’s agar as a
yeast at room temperature as well as at 35 degrees C. The yeast form
of this organism is readily converted to the hyphal form called germ
tubes in a test to be containing animal serum. Which of the following
is the most likely organism?
A. Candida albicans
B. Coccidioides immitis
C. Cryptococcus neoformans
D. Histoplasma capsulatum
E. Sporothrix schenckii
605. A gynecologist prescribes metronidazole to a women, this drug is not
likely to cover which organism?
A. Candida albicans
B. Trichomonas vaginalis
C. Giardia lamblia
D. Entameba histolytica
E. Bacteroides fragilis
606. Regarding Candida albicans which of the following are not true?
A. Is the most common species of Candida.
B. Appears as branching hyphae in the tissues.
C. Is normally found in oral cavity, gastrointestinal tract & vagina.
D. Is an opportunist pathogen.
E. Is not affected by normal bacterial flora.

607. The vaginal culture of a lady of 28 years showed non-pathogenic


bacteria, but on smear under microscope numerous bacilli were seen.
Which of the following right?
A. Gardenella vaginalis
B. E.coli
C. Lactobacillus
D. Proteus

Downloaded by AMAN SINGH (amansinghgkp8@gmail.com)


lOMoARcPSD|19252957

E. Pseudomonas

608. A woman complains of vulvar itching with pinkish vaginal discharge.


Which of the following is the most probably diagnosis?
A. Trichomonas vaginalis
B. N. gonorrhea
C. Lactobacillus
D. Chlamydia trachomatis
E. Hemophilus duceryi

609. Which subtype of E.Coli causes hemorrhagic colitis?


A. 0157:H7
B. 026:H1
C. Both of the above
D. None of the above
E. 057:H3

610. What is incubation period of typhoid?


A. 10-14 days
B. 2-6 days
C. 18-72 hours
D. 2-3 months
E. 20-25 days
611. Which of following is Vidal test?
A. Slide agglutination test
B. Flocculation test
C. Tube agglutination test
D. Complement fixation test
E. ELISA

612. Which of the following can’t be detected labile toxin of E. coli?


A. Into infant rabbit bowel
B. Into adult rabbit' skin
C. Intra gastrically into infant mouse
D. Into tissue culture of Chinese hamster ovary cells
E. Into YI mouse adrenal cells

613. What is true about Vidal test?


A. Vidal test confirmative in endemic area
B. Antibiotic treatment does not alter Vidal test
C. Previous infection affects Vidal test
D. Does not alter with prior vaccination
E. Vidal test is not confirmative

614. Which type of E.coli caused traveller's diarrhea?

Downloaded by AMAN SINGH (amansinghgkp8@gmail.com)


lOMoARcPSD|19252957

A. Enterotoxigenic E. coli
B. Enteropathogenic E. coli
C. Enteroinvasive E. coli
D. Enterohemorrhagic E. coli
E. Enteroaggregative E. coli

615. What is true about typhoid ?


A. Incubation period 3-6 weeks
B. Chronic carrier is 10-15%
C. Vidal test is specific
D. Vi polysaccharide of bacterial cell used for vaccination
E. Absence of flagella

616. What is the correct statement about Vidal test?


A. Only O antigen is used
B. Is a tube agglutination
C. Any antibody titre is diagnostic
D. Antibody appears after 1-10 days of fever
E. Slide agglutination

617. For which of following Vidal test is done?


A. Typhoid fever
B. Salmonela
C. Brucellosis
D. Tularemia
E. All

618. Which of following are microorganisms that enter freshly laid eggs?
A. Salmonella
B. Brucella
C. Shigella
D. Vibrio cholerae
E. Staphylococcus

619. Which of the following is not Non-lactose fermentate?


A. Shigella sonnei
B. Shigella dysenteriae
C. Shigellaflexneri
D. Shigella boydii
E. Salmonella

620. Which E.coli caused persistant diarrhea?


A. EAEC
B. EIEC
C. ETEC

Downloaded by AMAN SINGH (amansinghgkp8@gmail.com)


lOMoARcPSD|19252957

D. EPEC
E. EHEC

621. Which of the following statement regarding Shigella dysenteriae type I


is true?
A. It can lead to hemolytic uremic syndrome
B. It produces an invasive enterotoxin
C. It is an facultative aerobes
D. It is MR negative
E. Motile
622. On the basis of ability to ferment of what Shigella are divided into sub
group?
A. Lactose
B. Maltose
C. Fructose
D. Mannitol
E. Glucose

623. Which of the following is true about Enteropathogenic E. coli?


A. Causes diarrhea in infants
B. Acts by invasion of intestinal epithelial cells
C. Adults are mostly affected
D. Affects immunocompromised host
E. It can lead to hemolytic uremic syndrome
624. Which of the following is true regarding Salmonella infection?
A. Urine culture is +ve in 1st week
B. Stool culture is +ve in 1st week
C. Blood culture is +ve in 3-7 days
D. Widal test is +ve in 1st week
E. Blood culture is +ve in 20days

625. Which antigen blocks the agglutination of salmonella by 0 antiserum?


A. H.
B. Fimbriae
C. Vi
D. О
E. К
626. Which of the following are not true regarding typhoid ?
A. Urinary carriers are more dangerous
B. Vi ab is used for detecting carrier
C. Vi is seen in normal population
D. Urine carrier is associated with anomalies
E. O antigen is seen in normal population
627. Which toxin is not mediated by C-AMP?
A.V. cholera 01
B. Heat stable E. coli toxin
C.Heat labile E. coli toxin
D.V. cholera 0137
E. Shigella toxin
628. Which of the following caused traveller's diarrhea?
A.Shigella
B.E. coli
C. E. histolytica
D. Giardiasis
E. Cyclospora

Downloaded by AMAN SINGH (amansinghgkp8@gmail.com)


lOMoARcPSD|19252957

629. Which of the following E. coli gives pink color?


A. Chocolate agar
B. L J medium
C. MacConkey's medium
D. Saline broth
E. Blood agar

630. Which E.coli cause watery diarrhea in children?


A. ETEC
B. EPEC
C. EIEC
D. EAEC
E. EHEC
631. Which of following performed toxin is not important in food
poisoning?
A. Staph aureus
B. Clostridium botulism
C. ETEC
D. B. Cereus
E. Salmonella
632. What is true in E. coli true?
A. ETEC is invasive
B. EPEC acts via cAMP
C. Pilli present in uropathogenic type .
D. ETEC causes HUS
E. Vi antigen

633. A child with fever with RBCs and pus in stools, What can be
causative organism?
A. ETEC
B. EHEC
C. EPEC
D. EAEC
E. EIEC
634. Which of following can be recommended as transport medium for
stool specimen suspected to contain enteric pathogens?
A. Arnie's medium
B. Buffered glycerol saline medium
C. MacConkey medium
D. Blood agar
E. Chocolate agar
635. Which of the following are not true about V. cholerae 0139?
A. Clinical manifestations are similar to Ol El tor
B. First discovered in Chennai
C. Produces 0139 lipopolysaccharide
D. Epidemiologically indistinguishable from Ol El tor
E. Agglutination test is negative
636. Which of the following bacteria acts by increasing AMP?
A. Vibrio cholerae
B. Staphylococcus aureus
C. E. coliheat stable toxin
D. Salmonella
E. Shigella
637. Which of following not true about vibrio 0139?
A. Can cause disease in distinguishable from E-, or clinically
B. First isolated in Chennai
C. Has O polysaccharide capsule
D. Antibody to V. cholerae is not protective against 0139
E. Produces 0139 lipopolysaccharide

Downloaded by AMAN SINGH (amansinghgkp8@gmail.com)


lOMoARcPSD|19252957

638. How does cholera toxin acts in the small intestine?


A. ADP ribosylation of G regulatory protein
B. Inhibition of adenyl cyclase
C. Activation of GTPase
D. Active absorption of NaCl
E. None of them

639. Which antibiotic treatment of choice for treating cholera in an adult is a single
dose of?
A. Tetracycline
B. Cotrimoxazole
C. Doxycycline
D. Furazolidone
E. Penicilline
640. What is effect of cholera toxin is mediated via the stimulation of
following second messenger?
A. cAMP
B.cGMP
C. Calcium-calmodulin
D. Acetylcholine
E. Activation of GTPase
641. Which is the selective media for vibrio?
A. TCBS
B. Skirrow medium
C. Sruart
D. MYPE
E. MacConkeys medium
642. What is the best suited medium for Vibrio cholerae?
A. Thayer martin
B. TCBS medium
C. Scirrow medium
D. Loeffler's medium
E. Blood agar
643. Which endotoxin of the following gram-negative bacteria does not play any
part in the pathogenesis of the natural disease?
A. Escherichia coli
B. Klebsiella sp.
C. Vibrio cholerae
D. Pseudomonas aeruginosa
E. Brucellosis
644. What is the function of Cholera toxin?
A. Increases the levels of intracellular cyclic GMP
B. Acts through the receptor for opiates
C. Causes continued activation of adenylate cyclase
D. Inhibits the enzyme phosphodiesterase
E. Break down of WBC
645. What is the drug of choice for treating cholera in pregnant women?
A. Tetracycline
B. Doxycycline
C. Furazolidone
D. Cotrimoxazole
E. Penicilline
646. Which of the following is the drug of choice for chemoprophylaxis of

Downloaded by AMAN SINGH (amansinghgkp8@gmail.com)


lOMoARcPSD|19252957

cholera?
A. Tetracycline
B. Doxycycline
C. Furazolidone
D. Cotrimoxazole
E. Vancomycin
647. What is true about V. cholerae?
A. One attack of V. cholerae gives life-long immu-nity
B. Affects adults and children with equal propensity in nonepidemic regions
C. In between epidemics, carrier states maintain the organism
D. Pathogenicity of 0-139 vibrio is due to O antigen
E. Non motile
F.
648. Which of the following statements about El-Tor Vibrios are not true?
A. Human are the only reservour
B. Can survive in ice cold water for 2-4 weeks
C. Killed by boiling for 30 seconds
D. Enterotoxin can have direct effects on other tissues besides intestinal
epithelial cells.
E. Has O polysaccharide capsule
649. Which toxin acts by ADP ribosylation?
A. Botulinum toxin
B. Pertussis
C. Diphtheria toxin
D. Shigella toxin
E. V. cholera
650. What is Cholera transmission?
A. Food transmits
B. Vaccination gives 90% efficiency
C. Air
D. Chlorination is not effective
E. Blood
651. In patient presenting with diarrhea due to vibrio cholera, which of the
following will be present?
A. Abdominal pain
B. Presence of leukocytes in stool
C. Fever
D. Headache
E. Occurrence of many cases in the same locality
652. What is true about epidemiology of cholera?
A. Chemoprophylaxis is not effective
B. Boiling of water cannot destroy the organism
C. Food can transmit the disease
D. Vaccination give 90% protection
E. Can transmit by air
653. A stool examination was carried out which showed organism with darting
motility. Which of the following organism may be in stool?
A. V. cholerae
B. Shigella
C. Salmonella
D. Corynebacteria
E. E. coli
654. Which of the following about cholera is true?
A. Invasive
B. Endotoxin is released
C. Recent infections in India are of classical type
D. Vibriocidal antibody titer measures prevalence
E. Gram +ve

Downloaded by AMAN SINGH (amansinghgkp8@gmail.com)


lOMoARcPSD|19252957

655. What is the function of B subunit of cholera toxin?


A. ADP ribosylation of G protein
B. To bind GM1 ganglioside receptor
C. To stabilize cholera toxin
D. To increase cGMP
E. To bind cAMP
656. Which of following belong to vibrio cholera?
A. Is strongly anaerobic
B. Grows best at 25°C
C. Has marked tolerance of alkaline pH
D. Grows over a wide range but best at a slight acid pH
E. Obligate anaerobic
657. Which of the following has shortest incubation period?
A. Plague
B. Cholera
C. Measles
D. D. Typhoid
E. E. Mumps

658. Which of following doesn`t have peritrichous flagella?


A. Bacillus
B. Vibrio cholera
C. E.coli
D. Clostridium
E. Salmonella
659. By which of following Cholera toxin acts?
A. Na+ K+ ATPase inhibition
B. Adenylate cyclase stimulation
C. Opening of chloride channel
D. Stimulation of Ca++ channel
E. None

660. What is true about Vibrio cholera?


A. Disease more common in woman
B. Classical vibrio protect against development of 0.13 - strain disease
C. Ej-tor is more milder than classical
D. Erythronycin is used in treatment
E. Airborne disease

661. Which of the following is transport media for cholera?

A.VR medium
B. LZ medium
C. Bile salt agar
D. TCBS
E. Carry-Blair medium

662. Which of the following statement is true about: Vibrio cholera?


A. There is no natural reservoir
B. Transported in alkaline peptone water medium
C. Halophilic
D. Oxidase negative

Downloaded by AMAN SINGH (amansinghgkp8@gmail.com)


lOMoARcPSD|19252957

E. Gram +ve
663. Which of the following statements about cholera are not true?
A. O and H antigens measure carrier state
B. Culture medium is TCBS agar
C. Produces indole and reduces nitrate
D. Synthesize neuraminidase
E. Non-motile
664. Which of the following statements are not true for El- tor cholera?
A. Infection is mild and asymptomatic
B. They are resistant to polymyxin-B unit dike
C. Chronic carriers are common
D. Secondary attacks rate is high in families
E. Fecal-oral transmission

665. Ogawa Inawa and Hikojima are the serotypes of which bacteria?
A. Yersina
B. Vibrio cholerae
C. E Coli
D. Salmonella typhi
E. Shigella
666. What is the selective plating medium for V. cholera?
A. Carry-Blair medium
B. TCBS agar
C. VR medium
D. MacConkey medium
E. Blood agar
667. What is not true about cholerae?
A. Incubation period range from 1-5 days
B. Produces isotonic diarrhea
C. Cholera toxin plays principal role
D. Antibodies 01 cholera provides protection against 0139 serotype also
E. Motile
668. Which organism grows in alkaline pH?
A. Vibrio
B. Klebsiella
C. Pseudomonas
D. E.coli
E. Salmonella
669. Which of following doesn`t cause invasive infection?
A. V. cholera
B. Neisseria
C. Streptococci
D. D. H. influenza
E. E. Candida
670. What is true about cholera?
A. Gram negative rod
B. Associated with fever
C. Causes painful watery diarrhea
D. It is an achlorhydria which renders an individual susceptible to disease
E. Gram positive rod
671. Which are not true statement regarding pertussis?
A. Secondary attack rate averages 90% in unimmunized contacts
B. Incubation period is around 14 days
C. Erythromycin is the drug of choice
D. Can affect people of any age
E. Main source of infection is chronic carriers

Downloaded by AMAN SINGH (amansinghgkp8@gmail.com)


lOMoARcPSD|19252957

672. A veterinary doctor had pyrexia of unknown origin. His blood culture in
special laboratory media was positive for gram-negative short bacilli which
was oxidase positive. Which one of the following is the likely organism
grown in culture?
A. Pasturella spp.
B. Francisella spp.
C. Bartonella spp.
D. Brucella spp.
E. Salmonella spp.
673. A farmer presenting with fever off and on for the past 4 years was
diagnosed to be suffering from chronic Brucellosis. Which of the following
serological tests wouldnotbehelpful in the diagnosis at this state?
A. Standard agglutination test
B. 2-mercapto-ethanol test
C. Complement fixation test
D. Coomb's test
E. PCR

674. Brucellosis can’t be transmitted by which of the following modes?


A. Contact with infected placenta
B. Ingestion of raw vegetables from infected farms
C. Person to person transmission
D. Inhalation of infected dust or aerosol
E. By milk
675. Brucella is transmitted by which of the following means?
A. Through placenta of animals
B. Person to person transmission
C. Aerosol
D. Eating uncooked food
E. By inhalation
676. A farmer rearing sheep, presented with complaints of fever and weakness
for the last one month. There is generalised lymphadenopathy. There was
also associated hepatomegaly Biopsy of liver showed non-caseating
granuloma. Which of the following is most likely infection?
A. Yersinia pesitis
B. Brucella canis
C. Francisella tularensis
D. Brucella melitensis
E. Bacillus anthrasis
677. Regarding Brucellosis, which of these are not true?
A. Man to man transmission
B. A zoonosis
C. Blood cultures used in diagnosis
D. Transmitted via animal products
E. By air
678. Which is not true about brucella?
A. B. abortus is capnophilic
B. Transmitted by aerosol can occur occasionally
C. Paesturisation destroys it
D. 2 ME is used to detect IgA
E. Zoonosis
679. Brucella melitensis is found in which animal?
A. Cattle
B. Camel
C. Sheep
D. Goat
E. Reindeer

680. Which of the following is not virulence factor of pertussis?

Downloaded by AMAN SINGH (amansinghgkp8@gmail.com)


lOMoARcPSD|19252957

A. Tracheal cytotoxin
B. Pertusis toxin
C. Capsule
D. Pertactin
E. FHA

681. For pathogenicity testing of which bacteria mouse is used?


A. M. Tuberculosis
B. Bordetella pertussis
C. C. diptheriae
D. Brucella
E. E.coli
682. Which organism caused whooping cough?
A. B. pertussis
B. H. influenzae
C. Pneumococcus
D. Meningococcus
E. Streptococcus
683. Which does not have a known animal reservoir?
A. Brucella melitensis
B. Bordettela pertussis
C. Pasturella multocida
D. Francisella tularensis
E. Yersinia pestis
684. Which organism caused Chancroid?
A. H. ducreyi
B. T. pallidum
C. Gonococcus
D. HSV
E. B.melitensis
685. Lice are not the vectors of which desease?
A. Relapsing fever 4
B. Q fever
C. Trench fever
D. Epidemic typhus
E. Scarlet fever
686. What is not true about scrub typhus?
A. Causative organism is R.TSUTSUGAMOSHI
B. Vector is trombiculide mite
C. Adult female feeds on vertebrate hosts
D. Tetracycline is the drug of choice
E. Regional lymphadenopathy
687. A patient complained of chills and fever following a
louse bite 2 weeks before. He had rashes all over the
body and was delirious at the time of presentation to
the hospital and subsequently went into coma. A provi-
sional diagnosis of vasculitis due to rickettsial infection
was made. Which one of the following can be the causa-
tive agent?
A. Rickettsia typhi
B. Rickettsia rickettsiae
C. Rickettsia prowazekii
D. Rickettsia akari
E. B.melitensis
688. Which of the following statement are not true regarding
Q fever?
A. It is a zoonotic disease

Downloaded by AMAN SINGH (amansinghgkp8@gmail.com)


lOMoARcPSD|19252957

B. Human disease is characterized by an interstitial


pneumonia
C. No rash is seen
D. Weil Felix reactions is very useful for diagnosis
E. Cannot cultivated on solid media
689. Which of the following statements is true about endem-
ic typhus?
A. Is caused by R. rickettsii
B. Is transmitted by bites of fleas
C. Has no mammalian reservoir
D. Can be cultured in chemical defined culture medium
E. Can transmited by air

690. What is true regarding endemic typhus?


A.Man is the only reservoir of infection
B. Flea is a vector of the disease
C. The rash developing into eschar is a characteristic presentation
D. Culture of the etiological agent in tissue culture is a diagnostic
E. Cultivation on agar
691. Which of the following is the etiological agent of Rocky Mountain
spotted fever?
A. R. rickettsii
B. Rochalimae quintana
C. R. tsutsugamushi
D. Coxiella bumetti
E. Moraxella
692. Regarding Chlamydia infection of the eyes, what is not true statement?
A. Mostly asymptomatic
B. Can be cultured
C. Inclusion conjunctivitis is an acute ocular infection caused by sexually
transmitted C. trachomatis strains (usually serovars D through K)
D. Penicillin is the treatment
E. Cultivated on culture media
693. In a patient with UTI; on smear, no bacteria are found on gram stain with
abundant pus cells, to demonstrate organism, which of the following is
useful?
A. McCoy cell line
B. Thayer martin medium
C. L. J. medium
D. Acid fast staining
E. Blood agar

694. Which of the following is not true regarding Chlamydia?


A. Has biphasic life
B. Elementary body is metabolically active
C. Reticulate body undergoes binary fission
D. Once it invades into cell it abates phagolysosomal fusion
E. Spherical shape
695. Which of the following does not cause Chlamydia?
A. Q. fever
B. Non-gonococcal urethritis
C. Trachoma
D. Salpingitis
E. Psittacosis
696. Which of the following is not a method of isolation of Chlamydia from
clinical specimes?
A. Yolk inoculation
B. Enzyme immunoassay

Downloaded by AMAN SINGH (amansinghgkp8@gmail.com)


lOMoARcPSD|19252957

C. Tissue culture using irradiated McCoy


D. Tissue culture using irradiated BHK cells
E. Animal inoculation
697. Which of the following statements are not true regarding Chlamydia?
A. Erythromycin is effective for therapy of Chlamydia, infections
B. Their cell wall lacks a peptidoglycan layer
C. They can grow in cell free culture media
D. They are obligate intracellular bacteria
E. Gram negative coccobacilli
698. A man presents to STD clinic with urethritis and urethetral discharge.
Gram stain shows numerous pus cell but no microorganism. Culture is
negative on routine laboratory media. Which is the most likely agent?
A. Chlamydia trachomatis
B. H. ducreyi
C. T. pallidum
D. N. Gonorrhoeae
E. Mycoplasma

699. In which of the following cell lines doesn’t grow Chlamydia?


A. HeLa
B. HeP2
C. McCoy
D. Human diploid fibroblast
E. HL

700. Isolation of Chlamydia from tissue specimen can be done by which of


following?
A. ELISA (Enzyme Linked Immune Assay)
B. Yolk sac inoculation
C. Direct Immunofluorescence Antibody test (DFA)
D. Polymerase Chain Reacton (PCR)
E. LCR
701. Which of the following statement is not true regarding Chlamydia
Trachomatis:
A. Elementary body is metabolically active
B. It is biphasic
C. Reticulate body divides by binary' fission
D. Inside the cell it evades phagolysome
E. By complex procces
702. A male patient with symptoms of urethritis. Examination reveals only pus
cells without any organism. What is the most likely cause?
A. Chlamydia trachomatis
B. H. ducreyi
C. Treponema pallidum
D. M. tuberculi
E. N.Gonorrhea
703. How mite transmits?
A. Scrub typhus
B. B. Trench fever
C. Endemic typhus
D. Epidemic typhus
E. Typhoid

704. Which of the following infection doesn`t cause Chlamydia trachomatis?


A. Pneumonia
B. Rhinitis
C. Conjunctivitis

Downloaded by AMAN SINGH (amansinghgkp8@gmail.com)


lOMoARcPSD|19252957

D. Urethritis
E. Sinusitis

705. Which of following doesn`t cause Chlamydia trachomatis?


A. Ophthalmia neonatorum
B. Lymphogranuloma venereum
C. Q-fever
D. Trachoma
E. Cervicitis
706. Which of following Chlamydia trachomatis does not cause?
A. Cervicitis
B. Salpingitis
C. Vulvitis
D. Urethritis
E. Ophthalmia

707. How Rickettsia prowazekii is transmitted?


A. Fleas
B. Mites
C. Tick
D.Louse
E.Mouse
708. Which of the following statement are not true about rickettsial?
A. These are transmitted by arthropod vectors
B. Eschar is not seen in RMSF
C. Well Felex reaction may be diagnostic
D. Cephalosporins are drug of choice
E. Cultivated on solid media
709. Chlamydia trachomatis is the causative agent for which trachoma:?
A. Is a yeast
B. Is an intracellular organism
C. Forms extracellular bodies which are diagnostic
D. Is never demonstrable in conjunctival scrapings
E. Free living organism
710. Which of the following is an obligate intracellular parasite?
A. Mycoplasma
B. Chlamydia trachomatis
C. Gram -ve bacilli
D. Gram +ve cocci
E. Staphyloccoccus
711. Transovarian transmission is seen in which of following?
A. Ricketssiae
B. Clamydia
C. Mycoplasm
D. Both
E. None
712. Inclusion body is seen in which of following?
A. Rickettsiae
B. Chlamydia
C. Mycoplasma
D. H. Pylori
E. Cryptococcus
713. Congenital syphilis can be diagnosed by which test?
A. IgMFTABS
B. IgGFTAABS
C. VDRL
D. TPI
E. Microscopy

714. False +ve VDRL is seen in which of following?

Downloaded by AMAN SINGH (amansinghgkp8@gmail.com)


lOMoARcPSD|19252957

A. Lepromatous leprosy
B. Infectious mononucleosis
C. HIV
D. Pregnancy
E. IV drug user
715. Chancre redux' is a clinical feature of which of following?
A. Early relapsing syphilis
B. Late syphilis
C. Chancroid
D. Recurrent herpes simplex infection
E.
716. A 23-year-old male had unprotected sexual intercourse
with a commercial sex worker. Two weeks later he de-
veloped a painless, indurated ulcer on the glans that ex-
udated clear serum on pressure. Inguinal lymph nodes
in both groins were enlarged and non-tender. What is the most ap-
propirate diagnostic test?
A. Gram's stain of ulcer discharge
B. Dark field microscopy of ulcer discharge
C. Giemsa stain of lymph node aspirate
D. ELISA for HIV infection
E. Agglutination test
717. Spirochaetes is among of which of the following?
A. Syphilis
B. Legionella
C. Mycoplasma
D. Brucella
E. Bordetalla
718. What is not true about primary chancre?
A. Painless ulcer
B. Painless lymphadenopathy
C. Covered with exudate
D. Indurated lesion
E. Organism can be cultured from exudative fluid
719. A male patient presented with agitation, restlessness and neck stiffness. He
had undergone treatment for penile ulcer - 3 years back. Lab investigation
used for prognosis of treatment?
A. TPI
B. VDRL
C. FTA-ABS
D. Dark field microscopy
E. Non-venereal Treponemas
720. A bacterial disease with 3 'R's i.e. rats, rice fields and rainfall is?
A. Leptospirosis
B. Plague
C. Melioidosis
D. Rodent bite fever
E. Brucellosis
721. Which of the following statements are not true regarding leptospirosis?
A. It is a zoonosis
B. Man is the dead end host
C. Man is an accidental host
D. Lice acts as reservoir of infection
E. Source of infection are rats, dogs
722. 20-year-old boy has admitted with history of fever, icterus, conjunctival
suffusion and hematuria for 20 days. Which of the following serological test
can be of diagnostic utility?

Downloaded by AMAN SINGH (amansinghgkp8@gmail.com)


lOMoARcPSD|19252957

A. Widal test
B. Microscopic agglutination test
C. Paul-Bunnell-test
D. Weil-Felix reaction
E. CFT
723. Which of the following not used for diagnosis of Leptospirosis?
A. Microscopic agglutination test
B. Dark field illumination
C. Macroscopic agglutination test
D. Weil felix reaction
E. Direct fruorocent
724. What is true regarding leptospirosis?
A. Rats are the only reservoirs
B. Fluroquinolones are the DOC
C. Person to person transmission rare
D. Hepatorenal syndrome occurs in 50% cases
E. Ampicilin is used
725. Which one of the following microorganisms uses anti-
genic variation as a major means of invading host de-
fenses?
A. Streptococcus pneumonia
B. Borrelia recurrentis
C. Mycobacterium tuberculosis
D. Listeria monocytogenes
E. Brucellamelitensis
726. Which of the following species of borrelia cause relapsing fever?
A. Borrelia recurrentis
B. Borrelia hermsii
C. Borrelia turicatae
D. Borrelia duttonii
E. B.Parkeri
727. What is incubation period of syphilis?
A. 1hour-5hour
B. 1 day-10 days
C. 24 hour-48 hour
D. 10 days -90 days
E. 5 days-10 days

728. Borrelia undergoes antigenic variation due to which following?


A. Plasmids
B. Transposons
C.Intrinsic mutation
D.All of the above
E.None of them
729. Who is reservoir of leptospira?
A. Cat
B. Dog
C. Rat
D. Monkey
E. Donkey
730. What is true about B. recurrentis?
A. Causes leptospirosis
B. Water borne disease
C. Vector borne disease
D. Transmitted by tick
E. Air borne disease
731. What is determined by microscopic examination of tuberculosis?
A. Serotype

Downloaded by AMAN SINGH (amansinghgkp8@gmail.com)


lOMoARcPSD|19252957

B. Biovar
C. Acid resistance
D. Delayed hypersensitivity
E. Virulence factors
732. What express bacteriological method is used to diagnose tuberculosis?
A. Cultivation on egg’s medium
B. Cultivation on citrate blood
C. Rabbit inoculation
D. Guinea pig inoculation
E. Cultivation on synthetic medium
Clinical Virology
733. What can be used for the cultivation of influenza virus?
A. Synthetic media
B. Blood agar
C. Cell culture
D. CMB
E. Nutrient broth
734. What is the mechanism of transmission of influenza infection?
A. Blood transfusion
B. Airborne
C. transmissive
D. Fecal-oral
E. Injection

735. What family does the parainfluenza viruses belong to?


A. Picornavirus
B. Togavirus
C. Orthomyxovirus
D. Paramyxoviruses
E. Herpes viruses
736. What form does the parainfluenza viruses have?
A. Threaded
B. Spherical
C. Rod-shaped
D. Cubic
E. Icosahedron
737. What is the parainfluenza virus genome represented?
A. Single-stranded RNA
B. Double stranded DNA
C. single stranded DNA
D. Double stranded RNA

Downloaded by AMAN SINGH (amansinghgkp8@gmail.com)


lOMoARcPSD|19252957

E. Defect DNA

738. What is the materials for research of influenza?


A. Excrement
B. Nasopharyngeal discharge
C. Sputum
D. Bile
E. Cerebrospinal fluid

739. By which method rapid diagnosis of influenza is made?


A. Immunofluorescence
B. Immunoblotting
C. Microcultures Price
D. Chicken embryo infection
E. Hemagglutination

740. In which reaction the identification of the influenza virus is carried out?
A. Inhibition of hemagglutination
B. Agglutination
C. Light microscopy
D. Agglutination
E. Ring Reciprocations

741. What is used for the specific treatment of influenza?


A. Antigrippin
B. Rimantadine
C. Immunoglobulin
D. Anatoxin
E. Sulfanilamide drugs

742. What is the mechanism of transmission of parainfluenza virus?


A. Transplacental
B. Sexual
C. Airborne
D. Fecal-oral
E. Transmissive

743. What is character for the clinical course of parainfluenza?


A. Tenezmy
B. Pneumonia with the development of croup
C. Diarrhea
D. Endometritis

Downloaded by AMAN SINGH (amansinghgkp8@gmail.com)


lOMoARcPSD|19252957

E. Orchit

744. What is used for the treatment and prevention of adenovirus infection?
A. Interleukin
B. Live and dead vaccines
C. Reverse Transcriptase Inhibitors
D. Antibiotics
E. Serum

745. For which family mumps virus belongs to?


A. Togavirus
B. Picornavirus
C. Orthomyxovirus
D. Herpes viruses
E. Paramyxoviruses

746. Through what does infection with parotitis virus occurs?


A. Water
B. Soil
C. Liquor
D. Saliva
E. Products

747. What is mumps virus in the body affects?


A. Lung tissue
B. Gastrointestinal tract
C. Parotid glands
D. Spleen
E. Hepatic tissue

748. What is the most characteristic clinical manifestations of parotiditis?


A. Angina
B. Orchit
C. Diarrhea
D. Cholecystitis
E. Gastroenterit

749. Whaich complications may develop in case of epidemic parotitis?


A. Hepatitis
B. Infertility
C. Endocarditis
D. Mastitis
E. Herpangina

Downloaded by AMAN SINGH (amansinghgkp8@gmail.com)


lOMoARcPSD|19252957

750. What immunity is formed after the postponed mumps?


A. Short term
B. Non-sterile
C. Antitoxic
D. Antibacterial
E. Antiviral

751. For which family does respiratory syncytial virus belongs to?
A. Picornavirus
B. Paramyxoviruses
C. Orthomyxovirus
D. Herpes viruses
E. Togaviruses

752. What is the transmission of contracting a PCV virus infection?


A. Airborne
B. Through the mouth
C. Fecal-oral
D. Sexual
E. Transplacental

753. How is rubella virus spread?


A. Alimentary
B. Transplant
C. Fecal-oral
D. Sexual
E. Transmissible

754. What is specific rubella prevention?


A. Compliance with the rules of personal hygiene and public hygiene
B. Healthy lifestyle
C. Vaccination
D. Introduction of immunoglobulin
E. Sanitary-educational work among the population

755. What clinical signs are noted for measles?


A. Low temperature
B. Liquid stools
C. Appearance of rash
D. Diarrhea
E. Cough

Downloaded by AMAN SINGH (amansinghgkp8@gmail.com)


lOMoARcPSD|19252957

756. What immunity is formed after measles transmission?


A. Lifelong
B. Short
C. Antibacterial
D. Non-sterile
E. Not formed

757. Increase in which antibody titer is diagnostic value in measles?


A. Ig A
B. Ig M
C. Ig G
D. Ig E
E. Ig D
758. What is character for measles?
A. Liver damage
B. Return of symptoms
C. Development of anemia
D. The cause of subacute sclerosing panencephalitis
E. Diarrhea
759. Where does smallpox is currently registered?
A. In the form of group outbreaks in different countries
B. In Somalia and Ethiopia
C. Liquidated all over the world
D. In the Arabian Peninsula
E. In Europe

760. Which of the following is not characteristic of variola viruses?


A. DNA-containing
B. Large
C. Difficult
D. Defective
E. Has a brick shape

761. Which material is used for laboratory diagnosis of smallpox?


A. Contents of vesicles, pustules
B. Crusts from the skin
C. Nasopharyngeal discharge
D. Blood
E. Blood sera

Downloaded by AMAN SINGH (amansinghgkp8@gmail.com)


lOMoARcPSD|19252957

762. How does the indication of variola virus when infecting tissue cultures
can be done?
A. Identification of Guarnieri cell
B. Haemadsorption Phenomenon
C. Formation of syncytia
D. Hemagglutination test
E. Inhibition hemagglutination

763. What is the main route of transmission for smallpox?


A. Contact Household
B. Air-dust-dust
C. Airborne
D. Alimentary
E. Sexual

764. Which feature is not of pathogenesis in smallpox?


A.Viremia
B. Defeat of the reticuloendothelial system
C. Education characteristic nodular blistering rash
D. Complications (blindness, deafness and others.
E. Affect saliva glands

765. What is characteristic for the variola virus?


A. Cytoplasmic inclusions of Babesh-Negri
B. Reproduction of the virus in the cells of the CNS
C. Lesions of the skin and mucous membranes with the formation of
vesicles and pustules
D. Penetration by alimentary
E. Teratogenic effect

766. What is the character of post-infectious immunity in smallpox?


A. Antibacterial
B. Lifelong
C. Transplacental
D. Short
E. Antitoxic

767. Who has been developed a method for the specific prophylaxis of
smallpox?
A. E. Jenner, 1796
B. A. Negri, 1840
C. D. Guarnieri, 1892
D. E. Paschen, 1907
E. Anders, 1949

Downloaded by AMAN SINGH (amansinghgkp8@gmail.com)


lOMoARcPSD|19252957

768. What character of rabies is an infection?


A. Acute
B. Anthroponotic
C. Slow
D. Intestinal
E. Endogenous

769. For what family the rabies virus belongs to?


A.Rhabdoviridae, genus Vesiculovirus
B. Flaviviridae, genus Flavivirus
C.Retroviridae, genus Lentivirus
D. Rabdoviridae, genus Lyssavirus
E. Arenaviridae, genus Arenavirus

770. What is not character of rabies virus ?


A. DNA-containing
B. RNA-containing
C. Spiral type of symmetry
D. Has supercapsid
E. Medium

771. What is fixed rabies virus?


A. Attenuating strain of street virus
B. Circulates among wild animals
C. Defective virus
D. Not cultivated in vitro
E. Transmitted from person to person

772. What the difference between a fixed rabies virus and street virus?
A. Genome
B. The degree of human virulence
C. Lack of Babesha-Negri cell
D. Inability to multiply
E. Resistance in environment

773. What animal is the most common sources of infection with rabies?
A. Dog
B. snake
C. birds
D. cattle
E. sheep

Downloaded by AMAN SINGH (amansinghgkp8@gmail.com)


lOMoARcPSD|19252957

774. What is the main transmission route for rabies?


A. Alimentary
B. Transmissible
C. Contact (when bitten by sick animals.
D. Airborne
E. Transplacental

775. On what does laboratory diagnosis of rabies is based?


A. Detecting increase in antibody titer
B. Determination of allergic alteration of the body
C. Detecting Babes-Negri bodies and Virus Antigen
D. Determination of IgG avidity
E. Intracerebral infection of mice

776. The reproduction of which virus are formed in brain cells Babes-Negri
bodies?
A. Fixed
B. Street
C. Smallpox
D. Most viruses
E. Herpes simplex

777. How long the incubation period for rabies?


A. 24 hours
B. 1 week
C. 3 month
D. Maybe up to 12 months
E. Few weeks

778. What is characteristic of the presence of Babesh-Negri bodies in the cells?


A. Congenital rubella
B. Chickenpox
C. Tick-borne encephalitis
D. Rabies
E. Acute sclerosing panencephalitis

779. What is character of post-infectious immunity in rabies?


A. Low-stressed
B. Short
C. Not studied (mortality 100 %.
D. Lifetime
E. Non-sterile

Downloaded by AMAN SINGH (amansinghgkp8@gmail.com)


lOMoARcPSD|19252957

780. On what the rabies prevention is based?


A. Planned immunization of people
B. Immunization of wild animals
C. Mass immunization of domestic and farm animals
D. Quarantine measures in the focus of infection
E. Planned destruction of animals that are natural reservoirs

781. What morphological feature of the rabies virus?


A. The presence of integrase
B. Bullet shape
C. Cultivation in the media
D. Icosahedron shape
E. Presence of DNA

782. What is the most common disease outcome in rabies?


A. Death
B. Euphoria
C. relapse
D. Reinfection
E. Diarrhea

783. What drug can only prevent the development of rabies?


A. Antiviral serum
B. Interferon
C. Immunoglobulin
D. Vaccine
E. Immunomodulator
784. What microorganisms are characterized by a disjunctive mode of
reproduction?
A. Bacteria
B. Fungi
C. Viruses
D. The simplest
E. All of the above

785. What does the term persistency mean?


A. Hematogenous spread of microorganisms
B. Autoimmune process
C. Immediate-type hypersensitivity
D. Long-term preservation without clinical manifestations
E. None of the above

786. What preparation is used for the specific prevention of rabies?


A. Live vaccine
B. Auto vaccine

Downloaded by AMAN SINGH (amansinghgkp8@gmail.com)


lOMoARcPSD|19252957

C. Anatoxin
D. Antitoxic serum
E. Killed vaccine

787. What infections is chickenpox by source of spread belongs to?


A. Anthroponotic
B. Intestinal
C. Particularly dangerous
D. Wounded
E. Zoonotic

788. A carriage of which viral infection can lead Herpes Zoster?


A. Simple Herpes
B. Infectious mononucleosis
C. Natural smallpox
D. Chicken pox
E. Lymphoma Burkitt

789. What complication can lead chicken pox?


A. Fecal-oral transmission
B. Natural foci
C. The occurrence of convulsive cough
D. Guarnieri bodies
E. The development of anomalies in the fetus

790. What infection are drug addicts, homosexuals, patients with hemophilia,
prostitutes at high risk for?
A. Rabies
B. Herpes
C. Poliomyelitis
D. HIV infection
E. Mouth

791. For what family HIV belongs to?


A. Orthomyxoviridae
B. Togaviridae
C. Adenoviridae
D. Poxviridae
E. Retroviridae

792. What shape have HIV virus particle?


A. Brick-shaped

Downloaded by AMAN SINGH (amansinghgkp8@gmail.com)


lOMoARcPSD|19252957

B. Filamentary
C. Spherical
D. Rod-shaped
E. Bullet

793. For which virus T-helper cells, macrophages, dendritic cells are targets?
A. Herpes
B. Hepatitis B
C. Rubella
D. Human immunodeficiency
E. Cytomegaly

794. What structure of HIVgenome?


A. Double-stranded DNA with a single-stranded region
B. Single-stranded linear RNA
C. Linear double-stranded DNA
D. Double RNA
E. Does not match submitted

795. Which virus have the affinity of viral glycoprotein gp120 with CD4 T-
helper receptors, a high level of antigenic variability, integration of the
virus nucleic acid into the T-helper chromosome?
A. Hepatitis A
B. Poliomyelitis
C. ECNO
D. Human immunodeficiency
E. Herpe

796. Which virus is the most common cause of sporadic viral encephalitis?
A. Japnese B encephalitis
B. Herpes simplex encephalitis
C. HIV encephalitis
D. Rubeola encephalitis
E. Meningo encephalitis

797. Which virus is the most causative agent a neonate develops encephalitis
without any skin lesions?
A. HSVI
B. HSVII
C. Meningococci
D. Streptococci
E. Staphylococci

Downloaded by AMAN SINGH (amansinghgkp8@gmail.com)


lOMoARcPSD|19252957

798. What is character for HSV-2 infection?


A. Primary infection is usually widespread
B. Recurrent attacks are due to reactivation of latent infection
C. Encephalitis can be caused by HSV-2
D. Newborn may acquire infection via the birth canal at the time of labour
E. Treatment is with acyclovir

799. How long does the infectivity of chickenpox last?


A. Till the last scab falls off
B. 6 days after onset of rash
C. 3 days after onset of rash
D. Till the fever subsides
E. 10 days after onset of rash

800. Which of the following complications can be caused by HHV5?


A. Chickenpox
B. Infectious mononucleosis
C. Exanthem subitum
D. Kaposi sarcoma
E. Bronchiolitis

801. Where does Varicella Zoster remains latent?


A. Trigeminal Gangilion
B. Macrophages
C. T-cells
D. B-cells
E. brain cells

802. Which of the following does not establish a diagnosis of congenital CMV
infection in a neonate?
A. Urine culture of CMV
B. IgG CMV antibodies in blood
C. Intra-nuclear inclusion bodies in hepatocytes
D. CMV viral DNA in blood by polymerase chain reaction
E. Virus isolation

803. Which of the following is not a pox virus?


A. Cow pox
B. Molluscum contagiosum
C. Smallpox
D. Chickenpox
E. Retro virus

804. By what procedure genital herpes simplex can be diagnosed?


A. Grams stain

Downloaded by AMAN SINGH (amansinghgkp8@gmail.com)


lOMoARcPSD|19252957

B. KOH preparation
C.Tzank smear
D. Acid fast stain
E. Albert stain

805. What is character for HSV-1?


A. Causes encephalitis
B. Affects T lymphocyte
C. Is always symptomatic
D. Is not reactivated
E. Is RNA virus

806. Which virus remains dormant but reactivate is?


A. Herpes simplex
B. Herpes zoster
C. EB virus
D. CMV
E. Influenza virus

807. Which virus causes Kaposi sarcoma?


A. EBV
B. Parvovirus
C. Herpes virus
D. Rotavirus
E. Poliovirus

808. Which of the following diseases can not caused by the Epstein Barr virus?
A. Hodgkins lymphoma
B. Burkitt's lymphoma
C. Infectious mononucleosis
D. Leukemia
E. Stomach cancer
809. For which family Epstien Barr virus belongs to?
A. It is a member of herpes virus family
B. It is a member of adenovirus family
C. it is a member of togavirus family
D. It is a member of retrovirus family
E. It is a member of hepatovirus family

810. What morphological feature is character for Adenovirus?


A. Double stranded DNA
B. Enveloped
C. Complex symmetry
D. Double stranded RNA

Downloaded by AMAN SINGH (amansinghgkp8@gmail.com)


lOMoARcPSD|19252957

E. Contain Gp 120

811. What is taxonomy classification of Varicella?


A. Enterovirus
B. Retrovirus
C. Poxvirus
D. Herpes virus
E. Poliovirus

812. To which group Epstein Barr virus belongs to?


A. Retrovirus
B. Herpes virus
C. RNA virus
D. Pox virus
E. Poliovirus

813. Which pox won’t grow in egg, animal cells?


A. Cow pox
B. Vaccinia
C. Variola
D. Molluscum
E. Influenza

814. Which of the following virus is a pox virus?


A. Variola
B. Coxsachie
C. ECHO
D. HSV
E. Zoster

815. What the mechanism of transmission for polio- virus?


A. It is transmitted by feco-oral route
B. transmitted by close contact
C. transmitted by blood
D. transmitted by air
E. transmitted by contaminated wounds

816. Which of the following is the most common cause of meningoencephalitis in


children?
A. Mumps
B. Arbovirus
C. HSV
D. Enterovirus
E. Influenza virus

817. Which one of the following clinical features are not associated with

Downloaded by AMAN SINGH (amansinghgkp8@gmail.com)


lOMoARcPSD|19252957

enteroviruses?
A. Mycocarditis
B. Pleurodynia
C.Herpangina
D. Hemorrhagic fever
E. Feco-oral route

818. What is character for pathogenesis of polio?


A. Paralytic polio is most common
B. Only one type exists
C. Increased muscular activity leads to increased paralysis
D. Polio drop given only in <3 years
E. Air-borne disease

819. Which type vaccine is used for polio virus?


A. Killed vaccine
B. Inactivated vaccine given IM
C. Chemical vaccine
D. Genetic engineered vaccine
E. Monovalent vaccine

820. Which type of polio is the most responsible for epidemics?


A. Type 1
B. Type 2
C. Type 3
D. Type 4
E. Type 5

821. Which clinical feature is not associated with Enterovirus?


A. Hemorrhagic fever
B. Pleurodynia
C. Herpangina
D. Aseptic meningitis
E. Paralysis

822. At what pH Enteroviruses are stable?


A. Stable at pH 5
B. Stable at pH 4
C. Stable at pH 3
D. Stable at pH 2
E. Stable at pH 1

823. What disease is caused by H5N1?


A. Bird flu virus
B. Vaccine for HIV

Downloaded by AMAN SINGH (amansinghgkp8@gmail.com)


lOMoARcPSD|19252957

C. Causative agent of Japanese encpehalitis


D. An eradicated virus
E. Types of poliovirus

824. What antigenic property influenza A virus have?


A. It has a double stranded segmented RNA
B. Pandemics are caused by antigenic drifts
C. Nucelocapsid antibody is not specific
D. Hemagglutinin and neuraminidase are strain specific
E. Only one serotype cause epidemics

825. In what virus segmented RNA is found ?


A. Influenza virus
B. Rabies vims
C. Herpes virus
D. Molluscum contagiosum virus
E. Adenovirus

826. In which infection incubation period longer than 10 days?


A. Influenza
B. Cholera
C. Plague
D. Chickenpox
E. Diptheria

827. Which of the following is not character for measles?


A. High secondary attack
B. Only one strain causes infection
C. Not infectious in prodromal period
D. Infections confer life-long immunity
E. RNA virus

828. What complication can be caused by mumps?


A. Meningoencephalitis can precede parotitis
B. Salivary gland involvement is limited to the parotids
C. The patient is not infectious prior to clinical parotid enlargement
D. Mumps orchitis frequently leads to infertility
E. Conjuctivitis

829. What is commonest complication of Mumps?


A. Orchitis and Oophoritis
B. Encephalitis
C. Pneumonia
D. Myocarditis
E.Cancer

Downloaded by AMAN SINGH (amansinghgkp8@gmail.com)


lOMoARcPSD|19252957

830. What complication mumps can cause in children?


A. Causes SSPE
B. Causes aseptic meningitis
C. Sublingual gland is involved commonly
D. Causes viremia
E. Causes tonsillitis

831. Which of the following is the 'Least common' complication of measles ?


A. Diarrhea
B. Pneumonia
C. Otitis media
D. SSPE
E. Encephalitis

832. Which virus can be detected by Negro body?


A. CMV
B. Rabies
C. Inclusion of herpes simplex
D. EBV
E. Polio

833. What includes Class II exposure in animal bites?


A. Scratches without oozing of blood
B. Licks on a fresh wound
C. Scratch with oozing of blood on palm
D. Bites from wild animals
E. Bites from wild animals

834. Which of the following morphological features is character for rabies virus?
A. It is double stranded - RNA virus
B. Contains a DNA-dependent RNA polymerase
C. RNA has a negative polarity
D. Affects motor neurons
E. DNA virus

835. From what factor rabies pathogenesis is depend on?


A. Incubation period depends on the site of bite
B. Incubation period depends on type of virus
C. Incubation period depends on type of animal
D. Incubation period depends on risk factors
E. Incubation period depends on age of patient

836. What is diagnostic feature of Rabies?


A. Guarneri bodies
B. Cowdry Body

Downloaded by AMAN SINGH (amansinghgkp8@gmail.com)


lOMoARcPSD|19252957

C. Negri bodies
D. Bollinger bodies
E. None of the above

837. Which disease is transmits by soft tick?


A. Relapsing fever
B. KFD (Kyasanur forest disease)
C. Tick typhus
D. Tularemia
E. Plague

836. Which of the following viral infections is transmitted by tick?


A. Japanese encephalitis
B. Dengue fever
C. Kyasanur forest disease (KFD)
D.Yellow fever
E. Rabies

837. Which one of the following is not arboviral disease?


A.KFD
B. West Nile Fever
C. Ganjam virus
D. RSV
E.Puumala virus

838. What is the most specific for Dengue diagnosis?


A. IgM ELISA
B. Tissue culture
C. CFT
D. Electron microscopy
E. Agglutination test

839. Which of the following is/are arboviral diseases?


A. Japanese encephalitis
B. Dengue
C. Yellow fever
D. Hand-foot-mouth disease
E. Rocky mountain spotted fever

840. What is not character for dengue fever?


A. Caused by 4 serotypes
B. SS DNA
C. Presents with Gp
D. Virus belongs to flavivirus genus
E. Contain segmented RNA

Downloaded by AMAN SINGH (amansinghgkp8@gmail.com)


lOMoARcPSD|19252957

841. What complications can be from congenital rubella?


A. Malignification
B. Meningitis and encephalitis
C. Deafness, cardiac malformation and cataract
D. Mental retardation
E. Weak immune system

842. Which one of the following methods is not used in laboratory diagnosis of
viral respiratory tract infections?
A. Detection of virus specific IgM antibodies in single serum specimen
B. Demonstration of viral antigens by indirect immunofluorescence assay in
nasopharyngeal washings
C. Isolation of viruses using centrifugation enhanced culture
D. Detection of viral hemagglutination inhibiting (HAI) antibodies in a single serum
specimen
E. Microscopy

843. When the risk of the damage of fetus by maternal rubella is maximum if
mother gets infected?
A. 6-12 weeks of pregnancy
B. 20-24 weeks of pregnancy
C. 24-28 weeks of pregnancy
D. 32-36 weeks of pregnancy
E. 39-40 weeks of pregnancy

844. Which one of the following viruses are not included in picorna group?
A. Encephalo myocarditis
B. HEV '
C. Foot and mouth virus
D. Polio virus
E. Echo

845. Which micro-organism can be used as weapon in biological terrorism?


A. Chickenpox virus
B. Rabies virus
C. Ebola virus
D. Influenza C virus
E. Human parvovirus

846. Which virus is not RNA virus?


A. Ebola virus
B. Vesicular stomatitis virus
C. Simian 40
D. Rabies

Downloaded by AMAN SINGH (amansinghgkp8@gmail.com)


lOMoARcPSD|19252957

E. Influenza

847. In what infection man is the only reservoir?


A. Rabies
B. Influenzae
C. Typhoid
D. Japanese B encephalitis
E.Plague

848. Which virus causes epidemic hemorrhagic conjunctivitis?


A. HSV
B. HZV
C. HIV
D. Picorna virus
E. Influenza

849. In which part of the nervous system Negri Bodies are commonly seen?
A. Hippocampus
B. Hypothalamus
C. Mamillary bodies
D. Cerebrum
E. Pons

850. What is the congenital rubella syndrome?


A. May be prevented by vaccination in early pregnancy
B. Causes intrauterine growth retardation
C. Not Causes cataracts
D. Causes deafness only if acquired before 16 weeks of gestation
E. DNA virus

851. How does KFD is transmitted?


A. By Fleas
B. By Mite
C. By Tick
D. By Mosquito
E. By Dog

852. For which group rubella vaccination is contra indicated?


A. Patient on immunosuppressant
B. Girl with Leukemia
C. Girls between 11-14 years
D. Pregnancy
E. Patient with immunodeficiency

Downloaded by AMAN SINGH (amansinghgkp8@gmail.com)


lOMoARcPSD|19252957

853. By what virus acute epidemic keratoconjuctivitis is caused?


A. Herpesvirus
B. Echo 51
C. Enterovirus 70
D. Enterovirus 72
E. Poliovirus

854. How many doses of cell culture vaccine is used for pre-exposure in Rabies?
A. 3 doses
B. 4 doses
C. 5 doses
D.6 doses
E. 7 doses

855. By which virus break bone fevers is caused?


A. Variola
B. Coxsackie
C. Arbo
D. Adenovirus
E. Poliovirus

856. Which one is not character for rabies pathogenesis?


A. 100% mortality
B. Spreads from periphery
C. Infects only the brain
D. Prophylactic immunization of people at Risk
E. Transmits by dog bite

857. By what procedure rabies diagnosis is done best?


A. Brain biopsy
B. Blood culture
C. Electron microscopy
D. Hemagglutination test
E. ELISA

858. What type of polio causes epidemics?


A. Type I virus
B.Type II virus
C. Combine of type I and types III virus
D. Combine of type II and type III virus
E. Type III

859. Recommended vaccines for rabies?


A. DPT

Downloaded by AMAN SINGH (amansinghgkp8@gmail.com)


lOMoARcPSD|19252957

B. BCG
C. MMR
D. HDCV
E. OPV

860. Which of the following is associated with acute hemorrhagic conjunctivitis?


A. Rhabdovirus
B. Enterovirus
C. Calcivirus
D. Echovirus
E. Poliovirus

861. What disease is caused by enteroviruses?


A.hemorrhagic conjunctivitis
B. Ac. follicular conjunctivitis
C. Posterior follicular conjunctivitis
D. Epidermic kerato conjunctivitis
E. Conjunctivitis

862. By which virus break bone fever is caused?


A. Variola
B. Coxsackie
C. Dengue
D. Adenovirus
E. Poliovirus

863. Which one of the following is character for influenza?


A. It is caused by an enveloped DNA virus
B. Laboratory studies may show neutropenia early in the course of disease
C. Primary infectious pneumonia is less common than secondary bacteria
pneumonia
D. Antiviral agents is given early prevents complications
E. Simple virus

864. Which virus lacking hemagglutinin and nuraminidase but have membrane
fusion protein?
A. RSV
B. CMV
C. HSV
D. Ebestein Barr virus
E. Poliovirus

865. Who first developed a vaccine for rabies?


A. Louis Pasteur
B. Robert Koch

Downloaded by AMAN SINGH (amansinghgkp8@gmail.com)


lOMoARcPSD|19252957

C. Edward Jenner
D. Landsteiner
E. Paul Ehrlich

866. What complication is commonly caused by Coxsackie group A?


A. Conjunctivitis
B. Aseptic meningitis
C. Hepatitis
D. Myocarditis
E. Poliomylitis

867. Which of the following viruses is composed of two distinct capsids enclosing
the double stranded RNA?
A. Adenovirus
B. Reovirus
C. Herpes virus
D. Myxovirus
E. Influenza

868. Which strain of dengue fever virus is the most virulent?


A.1
B.2
C. 3
D. 4
E. 5

869. What is antigenic shift?


A. Occurs every 2-3 years
B. Gradual change over time
C. Result from genetic recombination
D. Occurs in all influenza viruses
E. Transformation

870. Which group does Coxsackie virus belong to?


A. Herpes virus
B. Pox virus
C. Enterovirus
D. Myxovirus
E. Influenza

Downloaded by AMAN SINGH (amansinghgkp8@gmail.com)


lOMoARcPSD|19252957

871. By what virus Herpangina is caused?


A. Enterovirus
B. Rhinoviruses
C. Myxovirus
D. Rabies virus
E. Rotavirus

872. What is the basis of diagnosis of polio?


A. Detection of polio virus in stool
B. Serology
C. Limb wasting
D. AFP
E. Agglutination test

873. For isolation of which virus suckling mice is used?


A. Coxsachie Virus
B. Pox
C. Herpes
D. Adeno Virus
E. Mums virus

874. What complication is not classic triad of congenital rubella?


A. Cataract
B.Deafness
C. Retinitis
D. CHD
E. Cardiac defect

875. Which family include all oncogenic viruses containing RNA?


A. Picomaviridae
B. Herpesviridae
C. Retroviridae
D. Flaviviridae
E. Retrovirus

876. What is characteristic of prion?


A. Infectious proteins
B. Made up of bacteria and virus particles

Downloaded by AMAN SINGH (amansinghgkp8@gmail.com)


lOMoARcPSD|19252957

C. Nuclear material
D. Can be cultured in cell free media
E. Infectious LPS

877. What is chemical compound of prions?


A. DNA and RNA
B. DNA, RNA and proteins
C. RNA and proteins
D. Only proteins
E. Lipids and DNA

878. Which of the following is not prion associated disease?


A. Scarpie
B. Kuru
C. Creutzfeldt-Jakob disease
D. Alzheimer's disease
E. Fatal familial insomnia

879. True about Prion disease is all except:


A. Myoclonus is seen in 10% of the patients
B. Caused by infectious protein
C. Brain biopsy is diagnostic
D. Commonly manifests as dementia
E. Proteins

880. What is pathogenic action of prion?


A. Are virus coded
B. Cause misfolding of protein
C. Cleave protein
D. Defect in synthesis of protein.
E. Contain capsid

881. Which of the following is correct about prions?


A. Long incubation period
B. Destroyed by autoclaving at 121 °C
C. Nucleic acid present
D. Immunogenic.
E. Short incubation period
882. Which of the following infection agent lacks RNA?
A. Virus

Downloaded by AMAN SINGH (amansinghgkp8@gmail.com)


lOMoARcPSD|19252957

B. Staphylococci
C. Prions
D. Cryptococcus
E. Adenovirus

883. Which one is non-parenteral hepatitis?


A. Hepatitis A
B. Hepatitis B
C. Hepatitis C
D. Hepatitis D
E.Hepatitis E

884. Which of the following hepatitis virus is cultivable:


A. Hepatitis A
B. Hepatitis B
C. Hepatitis D
D. Hepatitis E
E. Hepatitis C

885. Which of the following hepatitis virus have significant perinatal transmission?
A. HEV
B. HCV
C. HBV
D. HAV
E. HDV

886. Which gene is coded reverse transcriptase of HBV?


A. C gene
B. S gene
C. P gene
D. X gene
E. E gene

887. What is a marker for the diagnosis of acute hepatitis B?


A. HBV DNA Polymerase
B. IgG anti HBc
C. Core antgen (HbcAg)
D. Anti-HbsAg
E. HBxAg

Downloaded by AMAN SINGH (amansinghgkp8@gmail.com)


lOMoARcPSD|19252957

888. What markers of hepatitis B should not be detected in chronic form?


A. Hbs Ag
B. IgM anti-HBCAg
C. Hbe Ag
D. Anti-HbsAg
E. Anti-Hbe Ag

889. What markers indicate recent hepatitis B disease?


A. HBsAg
B. IgM anti-HBcAg
C. Anti-HBe
D. Anti-HBs
E. Hbe Ag

890. Which of the following hepatitis virus is a DNA virus?


A. Hepatitis C virus
B. Hepatitis B virus
C. Delta agent
D. Hepatitis E virus
E. Hepatitis A virus

891. What should be given to the newborn to prevent neonatal infection if mother is
HbsAg positive?
A. Hepatitis B vaccine + immunoglobulins
B. Immunoglobulins only
C. Hepatitis B vaccine only
D. Immunoglobulins followed by vaccine 1 month later
E. Only vaccine

892. What can be found in blood serum after hepatitis B vaccination?


A. HBsAg
B. Anti-HBsAg
C. IgM Anti-HBc Ag and HBS Ag
D. IgM and IgG Anti-HBc Ag
E. Anti- HbeAg

893. What make it possible to detect acute hepatitis B in the early stages?
A. IgM anti-HBcab
B. HBsAg
C. IgG anti-HBcAb

Downloaded by AMAN SINGH (amansinghgkp8@gmail.com)


lOMoARcPSD|19252957

D. Anti-HBsAb
E. HBeAg

894. What is best epidemiological tool for investigation of hepatitis B?


A. Anti-HBsAg
B. Anti-HBcAg
C. Anti-HBeAg
D. HBcAg
E. HBtAg

895. In a patient only anti-HBsAg is positive in serum all other viral markers are
negative. What it indicates?
A. Acute hepatitis
B. Chronic active
C. Persistent carrier
D. Hepatitis B vaccination
E. Just infecting

896. If a patient was immunized with hepatitis B vaccine, which of the following is
seen in serum?
A. HBeAg
B. HBSAg
C. Anti-HBs antibody
D. Anti-HBe antibody
E. Anti-HBc antibody

897. Which of these is not a marker of active replicative phase of chronic hepatitis?
A. HBV DNA
B. HBV DNA polymerase
C. Anti-HBC
D. AST and ALT
E. Ig M anti Hbs Ag

898. Which one is not character for pathogenesis of hepatitis B?


A. Vertical transmission is more important than horizontal
B. Communicable period lasts for months
C. Virus can be found in blood 1 month before jaundice
D. Age of onset determines the prognosis
E. Transmission by blood

Downloaded by AMAN SINGH (amansinghgkp8@gmail.com)


lOMoARcPSD|19252957

899. Reverse transcriptase is a RNA dependent DNA polymerase. Which of these


use it?
A. Hepatitis A virus
B. Hepatitis B virus
C. Hepatitis E virus
D. Hepatitis C virus
E. Hepatitis D virus

990. What virus is the most common hepatotropic virus causing increased chronic
carrier state?
A. HEV
B. HAV
C. HBV
D. HCV
E. HDV

991. What specificity is characteristic for hepatitis C in diagnosis?


A. Antibody can’t be detected
B. Antibody to HCV may not be seen in acute stage
C. May be diagnosed only in chronic form
D. It cannot be cultured
E. May be diagnosed if it is cirrhosis

992. What is the morphology of HCV?


A. Enveloped RNA virus
B. Unenveloped RNA virus
C. Unenveloped positive strand RNA
D. Unenveloped negative strand RNA
E. DNA virus

993. A young pregnant woman presents with fulminant hepatic failure. What is the
most likely etiological agent is?
A. Hepatitis B virus
B. Hepatitis C virus
C. Hepatitis E virus
D. Hepatitis A virus
E. Hepatitis D virus

994. For which group during epidemic of hepatitis E, fatality is maximum?


A. Pregnant women

Downloaded by AMAN SINGH (amansinghgkp8@gmail.com)


lOMoARcPSD|19252957

B. Infants
C. Malnourished male
D. Adolescents
E. Adults

995. With which of the following of viral hepatitis infection in pregnancy, the
maternal mortality the highest?
A. Hepatitis A
B. Hepatitis B
C. Hepatitis C
D. Hepatitis E
E. Hepatitis D virus

996. Which of the following is calcivirus?


A. HEV
B. HBV
C. HCV
D. HAV
E. HDV

997. In which case a blood donor is not considered for safe transfusion, if he has?
A. Anti-HBS Ab +ve
B. Anti-HBS Ab and HBc Ag +ve
C. HbS Ag +ve and IgM anti-HBC +ve
D. Anti-HBe +ve
E. IgM anti Hbs Ag

998. Which of the following acute viral hepatitis infections has the highest risk of
progression to chronicity?
A.Hepatitis C
B. Hepatitis B
C. Hepatitis A
D. Hepatitis E
E. Hepatitis D

999. Which hepatitis virus is notorious for causing a chronic hepatitis evolving
cirrhosis?
A. Hepatitis C virus
B. Hepatitis B virus

Downloaded by AMAN SINGH (amansinghgkp8@gmail.com)


lOMoARcPSD|19252957

C. Hepatitis E virus
D. Cytomegalovirus
E. Hepatitis D virus

1000. What infection is the most effective vaccine for?


A. Hepatitis A
B. Hepatitis D
C. Malaria
D. Respiratory syncitial virus
E. Hepatitis C

Downloaded by AMAN SINGH (amansinghgkp8@gmail.com)

You might also like